Chemistry 12th Previous Year Question Paper 2019 (CBSE)

Chemistry

Set-I

Section – A

Q.1.Out of KCl and AgCl, which one shows Schottky defect and why? 

OR

Q.1.Why does ZnO appear yellow on heating?

Answer: KCl shows schottky defect because cation & anions are of similar size. 

OR

Answer: ZnO on heating loses oxygen leaving behind their electrons at that position due to electrons it appears yellow in colour.

 

Q.2. Arrange the following in decreasing order of basic character:

C6H5NH2, (CH3)3N, C2H5NH2

Answer: Decreasing order of basic character:

CH3CH2NH2 > (CH3)3N > C2H5NH2

 

Q.3. What type of colloid is formed when a solid is dispersed in a liquid ? Give an example. 

Answer: Sols are formed when a solid is dispersed in a liquid.

Example – Paints.

 

Q.4. Out of Chlorobenzene and Cyclohexyl chloride, which one is more reactive towards nucleophilic substitution reaction and why?

Answer: Cyclohexyl chloride is more reactive towards nucleophilic substitution reaction because the carbon bearing the chlorine atom is deficient in electron and seeks a nucleophile. In Chlorobenzene the carbon bearing the halogen is a part of an aromatic ring and is electron-rich due to the electron density in the ring.

 

Q.5. What is the basic structural difference between starch and cellulose? 

OR

Q.5. Write the products obtained after hydrolysis of DNA.

Answer: Starch consists of two components- amylose and amylopectin. Amylose is a long linear chain of α-D -(+)-glucose units joined by C1, C4 glycosidic linkage (α-link). Amylopectin is a branched-chain polymer of α-D-glucose units, in which the chain is formed by C1-C4 glycosidic linkage and the branching occurs by C1-C6 glycosidic linkage. On the other hand, cellulose is a straight-chain polysaccharide of β-D-glucose units joined by C1-C4 glycosidic linkage (β-link).

OR

Answer: Hydrolysis of DNA yields a pentose sugar (β-D-2deoxyribose), phosphoric acid and nitrogen-containing heterocyclic compounds called bases (Adenine, Guanine, Cytosine and Thymine).

 

Section – B

Q.6. Write balanced chemical equations for the following processes:

(a) Cl2 is passed through slaked lime.

(b) SO2 gas is passed through an aqueous solution of Fe (III) salt. 

OR

Q.6. (a) Write two poisonous gases prepared from chlorine gas.<br> (b) Why does the Cu2+ solution give a blue colour on reaction with ammonia?

Answer: (a) Cl2 is passed through slaked lime to give bleaching powder [Ca(OCl)2]

2Ca(OH)2 + 2Cl2 → Ca(OCl)2 + CaCl2 + 2H2O

(b) When SO2 gas is passed through a Fe(III) aqueous solution, Fe(III) is reduced to Fe(II) ion:

2Fe3+ + SO2 + 2H2O → 2Fe2+ + SO22- + 4H+

OR

Answer: (a) Two poisonous gases prepared from chlorine – Phosgene (COCl2) and tear gas (CCl3NO2).

(b) Nitrogen in ammonia has a lone pair of electrons, which makes it a Lewis base. It donates the electron pair and forms linkage with metal ions-

 

Q.7. Give reasons:

(a) Cooking is faster in a pressure cooker than in cooking pan.

(b) Red Blood Cells (RBC) shrink when placed in saline water but swell in distilled water. 

Answer: (a) Boiling points increase in increasing the pressure in case of liquids. Water used for cooking attains a higher temperature than the usual boiling temperature inside the pressure cooker due to the existing high pressure inside the pressure cooker vessel. This leads to a faster flow of water inside the vegetables or grains etc. resulting in faster cooking of food in a pressure cooker than in the cooking pan.

(b) Red blood cells shrink when placed in saline water because of exosmosis, i.e., water comes out from the cell to surrounding (more concentrated) to equate the concentration. Whereas, when placed in distilled water concentration within the cell becomes more than the surrounding, hence water comes inside and endosmosis takes place to equate the concentrations.

 

Q.8. Define the order of the reaction. Predict the order of reaction in the given graphs:

where [R]0 is the initial concentration of reactant and t1/2is a half-life.

Answer: It is defined as the sum of powers to which the concentration terms are raised in the rate law equation.

(a) In this graph, as t1/2 is independent of initial reactant concentration, it is a first-order reaction.

(b) In this graph, as tin is directly proportional to the initial concentration of reactant hence, it is a zero-order reaction.

 

Q.9. When FeCr2O4 is fused with Na2CO3 in the presence of air it gives a yellow solution of compound (A). Compound (A) on acidification gives compound (B). Compound (B) on reaction with KCl forms an orange coloured compound (C). An acidified solution of compound (C) oxidises Na2O3 to (D). Identify (A), (B), (C) and (D). 

Answer: 

 

Q.10. Write IUPAC name of the complex [Co(en)2(NO2)Cl]+. What type of structural isomerism is shown by this complex? 

OR

Q.10. Using IUPAC norms, write the formula for the following complexes:

(a) Hexaaquachromium (III) chloride

(b) Sodium trioxalatoferrate (III)

Answer: IUPAC name of [Co(en)2(NO2)Cl]+ is Chlorobis(ethane-1, 2-diamine)nitro cobalt(III).

This compound shows geometrical isomerism.

OR

Answer: (a) Hexaaquachromium(III) chloride- [Cr(H2O)6]Cl3

(b) Sodium trioxalatoferrate(III)- Na3[Fe(C2O4)3]

 

Q.11. (a) Although both [NiCl4]2- and [Ni(CO)4] have sp3 hybridisation yet [NiCl4]4 is paramagnetic and [Ni(CO)4] is diamagnetic. Give reason. (Atomic no. of Ni = 28).

(b) Write the electronic configuration of d5 on the basis of crystal field theory when

(i) ∆0 &lt; P and 

(ii) ∆0&nbsp;&gt; P [2]

Answer: (a) [NiCl4]2- is a high spin complex and there are two impaired electrons with 3d8 electronic configuration of a central metal atom, hence it is paramagnetic. Whereas in [Ni(CO)4] Ni is in zero oxidation state and contains no unpaired electrons, hence it is diamagnetic in nature.

(b) (i) Electronic configuration of d5 when ∆o &lt; P is given as t2g3 eg2

(ii) Electronic configuration of d5 when ∆o &gt; P is given as t2g5 eg0.

 

Q.12. Write structures of main compounds A and B in each of the following reactions: 

Answer: 

 

Section – C

Q.13. The following data were obtained for the reaction:

A + 2B → C

(a) Find the order of reaction with respect to A and B.

(b) Write the rate law and overall order of the reaction.

(c) Calculate the rate constant (k).
Answer: The reaction is A + 2B → C

(a) Find the order of reaction with respect to A and B.

(b) Write the rate law and overall order of the reaction.

(c) Calculate the rate constant (k).

Answer: The reaction is A + 2B → C

(a) It can be seen that when the concentration of A is doubled keeping B constant, then the rate increases by a factor of 4 (from 4.2 × 102- to 1.68 × 102-). This indicates that the rate depends on the square of the concentration of the reactant A. Also when the concentration of reactant B is made four times, keeping the concentration of reactant A constant, the reaction rate also becomes 4 times (2.4 × 102- to 6.0 × 103-). This indicates that the rate depends on the concentration of reactant B to the first power.

(b) So, the rate equation will be:

Rate = k[A]2[B]

Overall order of reaction will be 2 + 1 = 3.

(c) Rate constant can be. calculated by putting the values given.

4.2 × 102- M min-1= k (0.2)2(0.3) M

k = 0.0420.0123.5 min-1

 

Q.14. (a) Write the dispersed phase and dispersion medium of dust.

(b) Why is physisorption reversible whereas chemisorption is irreversible?

(c) A colloidal sol is prepared by the method given in the figure.

What is the charge of Agl colloidal particles formed in the test tube?

How is this sol represented? 

Answer: (a) In dust, the dispersed phase is solid particles and the dispersion medium is air (gas).

(b) Physisorption occurs only because of physical attractive forces such as van der Waals forces between molecules of adsorbate and adsorbent, hence that can be reversed on the application of bigger forces but chemisorption occurs due to the chemical reaction between molecules of adsorbate and adsorbent, and hence can’t be reversed.

(c) When KI solution is added to AgNO3 a positively charged sol results due to absorption of Ag+ ions from dispersion medium-AgI/ Ag+(positively charged).

 

Q.15. An element X with an atomic mass of 81 u has density 10.2 g cm3-. If the volume of the unit cell is 27 × 10-23 cm3, identify the type of cubic unit cell. (Given: NA = 6.022 × 10 mol-1.

Answer: 

Q.16. A solution containing 19 g per 1oo mL of K (M = 74.5 g mol-1) is isotonic with a solution containing 3 g per 100 mL of urea (M = 60 g mol-1). Calculate the degree of dissociation of KCl solution. Assume that both the solutions have the same temperature. 

Answer: Two solutions having the same osmotic pressure at a given temperature are called isotonic solution. Now in the given problem, the KCl and urea solutions are given to be isotonic.

Osmotic pressure π is given by the equation

π = (n2/V)RT,       where n2 = moles of solute,

 

                                              V = volume of solution in litres.

                                    Also, n2 = w2/M2,

                                 where W2 = grams of solute  and M2 = molar mass of solute.

The other given information is

The molar mass of KCl = 74.5 g mol-1

Weight of KCl, W2= 1.9 g,  V = 100 mL

So, for KCl

                 n = (w2/M2 × V)RT

                 nRTKCl = 1.9/(74.5 × 100) = 2.55 × 10-4

Now as the solutions are isotonic at the same temperature:

                        πRTKCl = πRTurea

Hence, substituting the values for urea:

          2.55 × 10-4 = 3/M2 × 100

         M2 = 117.6

So, the experimentally determined molecular weight of urea is found to be as 117.6, so the degree of dissociation can be given as:

Osmotic pressure (TT) = Experimentally determined.

So, Urea dimerized in the given experimental solution.

 

Q.17. Write the name and principle of the method used for refining of (a) Zinc, (b) Germanium, (c) Titanium.

Answer: (a) Distillation is used for refining zinc. As zinc is a low boiling metal, the impure metal is evaporated and the pure metal is obtained as a distillate.

(b) Zone refining is used for refining Germanium. This method is based on the principle that the impurities are more soluble in the melt than in the solid-state of the metal.

(c) Titanium is refined by van Arkel method. This method is used for the removal of oxygen and nitrogen present as an impurity. The crude metal is heated in an evacuated vessel with iodine to obtain metal iodide, which volatilizes being covalent. Later this metal iodide is decomposed through electrical heating to obtain the pure metal.

 

Q.18. Give reasons for the following:

(a) Transition metals form complex compounds.

(b) E0 values of (Zn2+/Zn) and (Mn2+/Mn) are more negative than expected.

(c) Actinoids show a wide range of oxidation states.

Answer: (a) Transition elements have partly filled d-orbitals due to which they have variable oxidation states which enables them to bind with a variety of ligands and hence form complex compounds.
(b) Oxidation of Zn to Zn2+ leads to a completely filled d10 configuration in Zn2+, making it more stable. Also, Mn/Mn2+ conversion leads to a half-filled stable d5 configuration of Mn2+ ion. Hence, E0 value for Zn/Zn2+ and Mn/ Mn2+ conversion has negative values.

(c) Actinoids show a wide range of oxidation states due to their partially filled f-orbitals and they have comparable energies as well.

 

Q.19. Write the structures of monomers used for getting the following polymers:

(a) Nylon-6

(b) Terylene

(c) Buna-N 

OR

Q.19. (a) Is [CH2-CH(C6H5)]n homopolymer or copolymer? Give reason.

(b) Write the monomers of the following polymers:

(c) Write the role of benzoyl peroxide in the polymerisation of ethene.

Answer: Structures of monomers

(a) Caprolactam is monomer of Nylon-6

 

Q.20. (a) Pick out the odd one from the following on the basis of their medicinal properties:

Equanil, Seconal, Bithional, Luminal

(b) What types of detergents are used in dishwashing liquids?

(c) Why is the use of aspartame limited to cold foods? 

OR

Q.20. Define the following terms with a suitable example of each:

(a) Antibiotics

(b) Antiseptics

(c) Anionic detergents

Answer: (a) ‘Bithionol’ is the odd one here, as it is an antiseptic whereas others are tranquilisers.

(b) Liquid dishwashing detergents are non-ionic type.

(c) Aspartame is an artificial sweetener which is unstable at cooking temperature hence its use is limited to cold foods.

OR

Answer: (a) Antibiotics: These are the compounds (produced by microorganisms or synthetically) which either inhibit the growth of bacteria or kill bacteria. Example: Penicillin.

(b) Antiseptics: These are the chemicals used to kill or prevent the growth of microorganisms when applied to the living tissues.

Example: Soframycin.

(c) Anionic detergents: These are sodium salts of sulfonated long-chain alcohols or hydrocarbons. In these, the anionic part of the molecule is involved in the cleansing action.

Example: Sodium lauryl sulphate.

 

Q.21. Among all the isomers of molecular formula C4H8Br, identify:

(a) the one isomer which is optically active.

(b) the one isomer which is highly reactive towards SN2.

(c) the two isomers which give the same product on dehydrohalogenation with alcoholic KOH. 

Answer:

(a) 2- Bromobutane is optically active as C-2 is a chiral carbon here having all the four different groups attached to it.

(b) 1-Bromobutane being primary alkyl halide is highly reactive towards SN2 reaction.

(c) 2-Bromo-2-methylpropane and 1-Bromo-2-methylpropane would give the same product after dehydrohalogenation.

 

Q.22. Complete the following reactions:

OR

Q.22. How do you convert the following:

(a) N-phenylethylamine to p-bromoaniline

(b) Benzene diazonium chloride to nitro-benzene

(c) Benzoic acid to aniline

Answer:

Answer: 

 

Q.23. (a) Give reasons:

(i) Benzoic acid is a stronger acid than acetic acid.

(ii) Methanal is more reactive towards nucleophilic addition reaction than ethanal.

(b) Give a simple chemical test to distinguish between propanal and propanone.

Answer (a) (i) Benzoic acid is a stronger acid than acetic acid because the benzoate anion (the conjugate base of benzoic acid) formed after loss of H+ is stabilized by resonance, whereas acetate ion (CH3COO) has no such extra stability. Hence, Benzoic acid has more tendency of losing proton compared to acetic acid hence more acidic.

(ii) Methanal is more reactive towards nucleophilic addition reaction than ethanal because in ethanal there is a methyl group attached to the carbonyl carbon (centre for nucleophilic attack) and +1 effect of the methyl group decreases the nucleophilicity of carbonyl carbon by increasing the electron density at carbonyl carbon.

(b) Propanal and propanone can be distinguished using Tollen’s reagent by silver mirror test. Propanal being an aldehyde reacts with Tollen’s reagent to give silver deposition whereas propanone being a ketone does not give the reaction.

 

Q.24. (a) What is the product of hydrolysis of maltose?

(b) What type of bonding provides stability to the α-helix structure of a protein?

(c) Name the vitamin whose deficiency causes pernicious anaemia. 

OR

Q.24. Define the following terms:

(a) Invert sugar

(b) Native protein

(c) Nucleotide

Answer: (a) On hydrolysis maltose gives two molecules of α-D-glucose.

(b) α-Helix structure of proteins is stabilized by hydrogen bonds between -NH group of each amino acid and -COOH group of amino acid at adjacent turn.

(c) Deficiency of Vitamin B12 causes pernicious anaemia.

OR

Answer: (a) Invert sugar: It is a mixture of glucose and fructose obtained after hydrolysis of sucrose. Sucrose is dextrorotatory, but after hydrolysis gives a mixture of dextrorotatory glucose and levorotatory fructose which outweighs in magnitude and hence the whole mixture becomes levorotatory hence the mixture obtained is called invert sugar.

(b) Native protein: a Protein found in a biological system with a unique three-dimensional structure and biological activity is called a native protein.

(c) Nucleotide: They are building blocks of DNA/RNA. These consist of a pentose sugar moiety attached to a nitrogenous base at V position and a phosphoric acid molecule at 5′ position.

Example: 

Section – D

Q.25. (a) The conductivity of 0.001 mol L-1 acetic acid is 4.95 × 10-5 S cm-1. Calculate the dissociation constant if ∧0m for acetic acid is 390.5 S cm2 mol-1.
(b) Write Nest equation for the reaction at 25°C:

2Al(s) + 3Cu2+ (aq) → 2 Al3+ (aq) + 3Cu (s) (c)

What are secondary batteries? Give an example. 

OR

Q.25. (a) Represent the cell in which the following reaction takes place:

2Al (s) + 3 Ni2+ (0.1M) → 2Al3+ (0.01M) + 3 Ni (s)

Calculate its emf if E0cell = 1.41 V.

(b) How does molar conductivity vary with increase in concentration for strong electrolytes and weak electrolytes? How can you obtain limiting molar conductivity (∧m0) for weak electrolyte?

Answer: (a) Conductivity ∧m of a solution is given by the following equation:

m = K/C

where k is the dissociation constant and c is the concentration of the solution.<br> Here, given.

Conductivity, k = 4.95 × 10-5 S cm-1<br> Limiting molar conductivity,<br> ∧0m = 390.5 S cm2 mol-1

Concentration,

c = 0.001 mol L-1 = 1 × 10-3mol L-1

Substituting the given values in above equation

Molar conductivity,

(b) Nernst equation for the given reaction can be written as

(c) A secondary battery can be recharged after use, bypassing current through it in the opposite direction so that it can be used again.

Example: The most important secondary cell is lead storage cell. It consists of a lead anode and a grid of lead packed with lead dioxide as a cathode. A 38% solution of sulphuric acid is used as an electrolyte.

OR

Answer: (a) The cell can be represented as

(b) For strong electrolytes, the molar conductivity is increased only slightly on dilution. A strong electrolyte is completely dissociated in solution and thus, furnishes all ions for conductance. However, at higher concentrations, the dissociated ions are close to each other and thus, the interionic attractions are greater. These forces retard the motion of the ions and thus, conductivity is low. With a decrease in concentration (dilution), the ions move away from each other thereby feeling less attractive forces from the counterions. This results in an increase in molar conductivity with dilution. The molar conductivity approaches a maximum limiting value at infinite dilution designated as ∧m0.

In the case of weak electrolytes as the solution of a weak electrolyte is diluted, its ionization is increased. This results in more number of ions in solution and thus, there is an increase in molar conductivity, also there is a large increase in the value of molar conductivity with dilution, especially near-infinite dilution. However, the conductance of a weak electrolyte never approaches a limiting value. Or in other words, it is not possible to find conductance at infinite dilution (zero concentration).

So, limiting molar conductivity for weak electrolytes are obtained by using Kohlrausch law, from the limiting molar conductivities of individual ions (λ0).

Kohlrausch law of independent migration of ions states that limiting molar conductivity of an electrolyte can be represented as the sum of the individual contributions of the anion and the cation of the electrolyte.

m0 = λ0+ + λ0

Q.26. (a) Give the equation of the following reactions:

(i) Phenol is treated with cone. HNO3.

(ii) Propene is treated with B2H6 followed by H2O2/OH-.

(iii) Sodium t-butoxide is treated with CH3Cl.

(b) How will you distinguish between butan-l-ol and butan-2-ol?

(c) Arrange the following in increasing order of acidity: Phenol, ethanol, water.

OR

Q.26. (a) How can you obtain Phenol from (i) Cumene, (ii) Benzene sulphonic acid, (iii) Benzene diazonium chloride?

(b) Write the structure of the major product obtained from the denitration of 3-methyl phenol.

(c) Write the reaction involved in Kolbe’s reaction.

Answer: (a) (i) Phenol is treated with conc. HNO3 to obtain 2,4,6-trinitrophenol picric acid.

(ii) Propene undergoes hydroboration-oxidation when treated with B2H6 followed by hydrogen peroxide in basic medium to give propan-1-ol.

(iii) Methyl tert-butyl ether is produced when sodium tert-butoxide is treated with methyl chloride.

(b) Butan-l-ol and Butan-2-ol can be distinguished using Lucas reagent (ZnCl2+HCl), where butan-2-ol would react with Lucas reagent in around 5 minutes to give a white precipitate of 2-chlorobutane, whereas butan-l-ol won’t give any reaction at room temperature.

(c) Increasing order of acidity can be given as Ethanol < water < phenol

OR

Answer: (a) (i) Phenol from cumene

(ii) Phenol from benzene sulphonic acid

(iii) Phenol from benzene diazonium chloride

(b) The combined influence of -OH and -CH3 groups determine the position of the entering groups, also the sterically hindered positions are not substituted.

(c) In Kolbe’s reaction phenol is reacted with CO2 in the presence of sodium hydroxide, followed by acidification, to give a carboxylic acid group on 2-position of phenol-

 

Q.27. (a) Account for the following:

(i) The tendency to show – 3 oxidation state decreases from N to Bi in group 15.

(ii) Acidic character increases from H2O to H2Te.

(iii) F2 is more reactive than CIF3, whereas ClF3 is more reactive than Cl2.

(b) Draw the structure of (i) XeF2 (ii) H4P2O7

OR

Q.27. (a) Give one example to show the anomalous reaction of fluorine.

(b) What is the structural difference between white phosphorous and red phosphorous?

(c) What happens when XeF6 reacts with NaF?

(d) Why is H2S a better reducing agent than H2O?

(e) Arrange the following acids in the increasing order of their acidic character: HF, HCl, HBr and HI

Answer: (a) (ii) Acidic character increases from H2O to H2Te due to decrease in E—H bond dissociation enthalpy down the group. Thus it becomes easy to lose proton going down the group.

(iii) F2 is more reactive than ClF3 because of the small size of fluorine atom F—F bond, bond dissociation enthalpy is low (thus is reactive).

Whereas ClF3 is more reactive than Cl2 because ClF3 is an interhalogen compound with weak Cl—F bond (compared to Cl—Cl bond) due to the difference in atomic sizes (hence ineffective overlap of orbitals).

(b) (i) Structure of XeF2 is linear.

(ii) Structure of H4P2O7.

OR

Answer: (a) Fluorine reacts with cold sodium hydroxide solution to give OF2.

2F2 (g) + 2NaOH (aq) → 2NaF (aq) + OF2 (g) + H2O(l)

(c) XeFg reacts with NaF as follows:

XeF6 + NaF → Na+[XeF7]

(d) Ability to reduce is judged by the ease with which an atom can donate its electrons to the species which is getting reduced. Now, the size of oxygen atom in H2O is smaller than that of Sulphur atom in H2S, due to which the lone pair of electrons on oxygen are more attracted by the oxygen nucleus, making it difficult to donate electrons (by oxygen compared to Sulphur, while in H2S the influence of the nucleus is less on lone pair of electrons of sulphur and hence, it can give away its electrons, easily compared to oxygen, and thus acts as a better reducing agent.

(e) The increasing order of acidic character can be written as

HF < HCl < HBr < HI


 

Chemistry 

Set-II

Section – A

Q.2. Arrange the following in increasing order of pKb values:

C6H5CH2NH2, C6H5NHCH3, C6H6NH2 

Answer: These can be arranged in increasing order of pKb values as follows:

C6H5CH2NH2 < C6H5NHCH3 < C6H5NH2

 

Q.3. What type of colloid is formed when a liquid is dispersed in a solid? Give an example. 

Answer: When a liquid is dispersed in a solid, a ‘gel’ is formed.

Example: Butter.

 

Q.4. Out of chlorobenzene and p-nitrochloro-benzene, which one is more reactive towards nucleophilic substitution reaction and why? 

Answer: p-Nitro chlorobenzene would be more reactive towards nucleophilic substitution reaction compared to chlorobenzene. In chlorobenzene the carbon bearing the halogen is a part of the aromatic ring and is electron-rich due to the electron density in the ring so it does not attract the nucleophile. The -NO2 substitution lessens the electron density on the benzene ring due to its electron-withdrawing nature, making the electron density on ringless compared to chlorobenzene, hence p-nitro chlorobenzene attracts nucleophiles better.

 

Section -B

Q.7. Give reasons:

(a) A decrease in temperature is observed in mixing ethanol and acetone.

(b) Potassium chloride solution freezes at a lower temperature than water. 

Answer: (a) Upon mixing molecules of ethanol and acetone have strong intermolecular attractions due to which heat is evolved from the reaction system and hence cooling of mixture is observed.

(b) Potassium chloride solution is a solution of non-volatile solute KCl and water solution. We know that, at the freezing point of a substance, the solid phase (here ice) is in dynamic equilibrium with the liquid phase. A solution freezes when its vapour pressure equals the vapour pressure of the pure solid solvent. Now, according to Raoult’s law when a non-volatile solid is added to the solvent (in this case it is KCl), its vapour pressure decreases and now it would become equal to that of solid solvent at a lower temperature. Thus, the freezing point of the solvent decreases.

 

Q.10. Define the following terms with a suitable example of each:

(a) Chelate complex

(b) Ambidentate ligand

OR

Q.10. Using IUPAC norms, write the formula for the following complexes:

(a) Tetraamminedichlorocobalt (III) chloride

 (b) Dibromobis (ethane-1,2-diamine) platinum (IV) nitrate

Answer: (a) Chelate complex: Chelate complexes are coordination or complex compound consisting of a central metal atom attached to a large molecule, called a ligand, in a cyclic or ring structure. The ligands are bi or polydentate i.e., they can attach to metal atom through two or more than two binding sites. An example of a chelate ring occurs in the ethylenediamine-nickel complex.

(b) Ambidentate ligand: Ligands which can ligate (attach to the metal atom) through two different atoms is called ambidentate ligand. One example of such ligand is NO2 , this can bind through both the atoms, nitrogen and oxygen.

OR

Answer: IUPAC names

Tetraamine Diaqua Cobalt(III)chloride- [Co(NH3)4(H2O)2]Cl3

(b) Dibromobis(ethane-1,2-diamine) platinum(IV) nitrate -[PtBr2(en)2](NO3)2.

 

Section – C

Q.13. (a) Write the dispersed phase and dispersion medium of milk.

(b) Why is adsorption exothermic in nature?

(c) Write Freundlich adsorption isotherm for gases at high pressure.

Answer: (a) Dispersed phase of milk is liquid and the dispersion medium of milk is liquid.

(b) During the process of adsorption molecules of adsorbate and adsorbent come closer to form physical or chemical bonds hence getting stabilized, in this process heat is evolved leading the overall process to be exothermic.

(c) Freundlich adsorption isotherm for gases at high pressure.

It is an empirical relationship between the quantity of gas adsorbed by a unit mass of solid adsorbent and pressure at a particular temperature:         x/m = kp1/2(n > 1)


Where x is the mass of the gas adsorbed on mass m of the adsorbent at pressure p, k and n are constants which depend on the nature of the adsorbent and the gas at a particular temperature.

The relationship is generally represented in the form of a curve x/m is plotted against the pressure. This curve always approaches saturation towards high pressure, thus indicating that at adsorption through increases with increase in pressure till a limit and at high pressures, no further adsorption is observed.

 

Q.15. Write the name and principle of the method used for the refining of

(a) Tin,

(b) Copper,

(c) Nickel. 

Answer: (a) Tin: It is refined through liquidation. In this method, a low melting metal like tin is made to flow on a sloping surface, where the higher melting impurities are left behind and the lower melting metal is collected at the sloping end.

(b) Copper: It is refined through electrolytic refining. Anode is made of impure copper and pure copper stripes are taken as the cathode. They are dipped in acidified solution of copper sulphate, as an electrolyte. The net result of electrolysis is the transfer of copper in pure form from the anode to the cathode and the impurities get deposited as anode mud.

(c) Nickel: It is refined through Mond’s process. In this process, Nickel is heated in a stream of carbon monoxide forming a volatile complex, nickel tetracarbonyl.

The carbonyl is subjected to a higher temperature so that it is decomposed giving the pure metal.

 

Q.16. Give a reason for the following:

(a) Transition metals show variable oxidation states.

(b) E0 value of (Zn2+/Zn) is negative while that of (Cu2+/Cu) is positive.

(c) Higher oxidation state of Mn with fluorine is +4 whereas with oxygen is +7. 

Answer: (a) Transition metals show variable oxidation states because their d-orbitals are incompletely filled and different arrangements of electrons are possible according to the chemical environment of metal ions. hence, the ions can occupy variable oxidation states.

(b) E° value for Zn/Zn2+ is negative because the conversion of Zn to Zn2+ gives it a completely filled d5 configuration and extra stability gained by Zn2+. Whereas conversion of Cu to Cu2+ does not give any extra stability, hence it has a positive E0 value.

(c) Mn has the highest oxidation state of +4 with fluorine but with oxygen, it is +7. This is due to the ability of oxygen to form multiple bonds with the metal ion, whereas fluorine being of small size and devoid of d-orbitals can’t form multiple bonds.

 

Q.19. Write the structures of monomers used for getting the following polymers:

(a) Nylon-6, 6

(b) Bakelite

(c) Buna-S 

OR

Q.19. (a) Write one example each of:

(i) Thermoplastic polymer

(ii) Elastomers

(b) Arrange the following polymers in the increasing order of their intermolecular forces:

Polythene, Nylon-6, 6, Buna-S

(c) Which factor provides crystalline nature to a polymer like Nylon?

Answer: (a) Monomers of Nylon-6,6 are adipic acid and hexamethylenediamine.

(b) Monomers of bakelite are phenol and formaldehyde:

(c) Monomers of Buna-S are 1,3-Butadiene and Styrene

OR

Answer: (a) (i) Example of thermoplastic polymer – polythene, polystyrene.

(ii) Example of elastomer – Neoprene.

(b) In increasing order of their intermolecular force, they can be arranged as:

Buna-S < Polythene < Nylon-6,6

(c) Strong intermolecular forces between the polymer molecules, such as hydrogen bonding lead to closed packed structure, thus imparting crystalline nature to the polymers.


 

Chemistry

Set-III

Section – A

Q.1. Out of Chlorobenzene and Cyclohexyl chloride, which one is more reactive towards nucleophilic substitution reaction and why? 

Answer: Cyclohexyl chloride is more reactive towards nucleophilic substitution reaction because the carbon bearing the chlorine atom is deficient in electron and seeks a nucleophile. In Chlorobenzene the carbon bearing the halogen is a part of an aromatic ring and is electron-rich due to the electron density in the ring.

 

Q.2. Arrange the following in decreasing order of solubility in water:

(C2H5)2NH, C2H5NH2, C6H5NH2 

Answer: Decreasing order of solubility in water is:

C2H5NH2 > (C2H5)2NH2 > C6H5NH2.

 

Q.3. What type of colloid is formed when a solid is dispersed in a gas? Give an example.

Answer: An aerosol is the type of colloid formed when solid is dispersed in gas.

Example: smoke and dust.

 

Q.5. What is the difference between amylose and amylopectin? 

OR

Q.5. Write the products obtained after hydrolysis of lactose.

Answer: Amylose is a long linear chain of α-D-(+)-glucose units joined by C1-C4 glycosidic linkage (α-link), whereas Amylopectin is a branched-chain polymer of α-D-glucose units, in which the chain is formed by C1-C4 glycosidic linkage and the branching occurs by C1-C6 glycosidic linkage.

OR

Answer: Galactose and Glucose are the products obtained after hydrolysis of lactose.

 

Section – B

Q.7. Give reasons: 

(a) An increase in temperature is observed on mixing chloroform and acetone.

(b) Aquatic animals are more comfortable in cold water than in warm water.

Answer: (a) A mixture of chloroform and acetone forms a solution with a negative deviation from Raoult’s law. This is because chloroform molecule is able to form a hydrogen bond with acetone molecule as shown by the following figure:

This decreases the escaping tendency of molecules for each component, and consequently, the vapour pressure decreases and the temperature of the solution is increased because of stability attained by the molecule by associating and releasing energy.

(b) The solubility of gases in liquids increases on decreasing temperature, hence cold water has more dissolved oxygen because of which aquatic species find themselves more comfortable in cold water as compared to hot water.

 

Q.10. Define the following terms with a suitable example of each:

(a) Polydentate ligand

(b) Homoleptic complex

OR

Q.10. Using IUPAC norms, write the formula for the following complexes:

(a) Potassium tri (oxalato) chromate (III)

(b) Hexaaquamanganese (II) sulphate.

Answer: (a) Polydentate ligands: Ligands with several donor atoms are called polydentate ligands. These can bond with the metal ion in a complex with the different donor atoms present in them.

Example: N(CH2CH2NH2)3.

(b) Homoleptic complex: Complexes in which a metal atom is bound to only one kind of donor groups, e.g., [Co(NH3)6]2+ are known as homoleptic complex.

OR

Answer: (a) K3[Cr(C2O4)3]

(b) [Mn (H2O)6] SO4.

 

Q.12. Write structures of main compounds A and B in each of the following reactions:

Answer:

 

Section -C

Q.14. (a) Write the dispersed phase and dispersion medium of butter.

(b) Why does physisorption decrease with increase in temperature?

(c) A colloidal sol is prepared by the method given in the figure. What is the charge on AgI colloidal particles formed in the test tube? How is this sol represented? 

Answer: (a) Butter is an example of ‘Gel’ type of colloid. Here the dispersed phase is liquid and dispersion medium is solid.

(b) Physisorption occurs because of physical attractive forces, like Vander Waals forces between molecules of adsorbate and adsorbent, hence that can be reversed on the application of bigger forces. Hence, when the temperature is increased, the movement of adsorbed molecules increases, resulting in disturbed attractive forces, detachment of adsorbed molecules from the adsorbent surface hence physisorption decreases.

(c) When the AgNO3 solution is added to KI, silver iodide, AgI, is precipitated. The precipitated silver iodide adsorbs iodide ions from dispersion medium and negatively charged colloidal sol results. It can be shown as AgI/I (negatively charged).

 

Q.17. Write the principle of the following:

(a) Hydraulic washing

(b) Chromatography

(c) Froth-floatation process 

Answer: (a) Hydraulic washing: This method of concentration of ores is based on the differences in gravity of the ore and gangue particles. It is a type of gravity separation. An upward stream of running water is used to wash the powdered ore. The lighter gangue particles are washed away and the heavier ores are left behind.

(b) Chromatography: Chromatography is a physical method of separation of a mixture in which the components to be separated are distributed between two phases, stationary and mobile phase. The stationary phase may be a solid or a liquid supported on a solid or a gel. The mobile phase may be either a liquid or a gas.

(c) Froth floatation process: Froth floatation is a physicochemical method of concentrating fine minerals. This process utilizes the difference in surface properties of valuable minerals and gangue (impurity) particles. For example, removal of gangue from sulphide ores.

 

Q.18. Give reasons for the following:

(a) Transition metals have high enthalpies of atomization.

(b) Manganese has a lower melting point even though it has a higher number of unpaired electrons for bonding.

(c) Ce4+ is a strong oxidising agent. 

Answer: (a) Transition element has high effective nuclear charge and a large number of valence electrons ((n-1) d electrons). So, as a result of the greater number of electrons participating, very strong metallic bonds are formed. As a result of the strong inter-atomic metallic bonding, the transition metals have high enthalpies of atomization.

(b) Manganese has a lower melting point even though it has a higher number of impaired electrons for bonding. Melting point depends on the intermolecular or interatomic forces. Stronger the forces, the higher the melting point. In Mn there is half-filled 3d subshell (3d5 configuration) which makes it stable and hence, it does not make additional covalent bonds with nearby atoms hence, it has less melting point.

(c) Ce4+ is a strong oxidising agent because Ce4+ oxidizes others and itself gets reduced to the common and preferred 3+ oxidation state of lanthanide elements.

 

Q.19. Write the structures of monomers used for getting the following polymers:

(a) Novolac

(b) Neoprene

(c) Buna-S 

OR

Q.19. (a) Write on example each of

(i) Cross-linked polymer

(ii) Natural polymer

(b) Arrange the following in the increasing order of their intermolecular forces:<br> Terylene, Buna-N, Polystyrene

(c) Define biodegradable polymers with an example.

Answer: (a) Novolac is the polymer of 2-hydroxymethyl phenol which is obtained by reaction of phenol and formaldehyde.

Answer: 

(b) Increasing order of their molecular forces:

 Buna-N < Polystyrene < Terylene

(c) Biodegradable polymer: These are synthetic polymers designed so as to contain functional groups similar to ones present in biopolymers. These are thus easily degraded by environmental degradation process hence, known as Biodegradable polymers.

Example: Poly (β-hydroxybutyrate-co-β-hydroxy valerate (PHBV).

 

Q.23. (a) Write the product when D-glucose reacts with Br2aq.

Answer: 

Biology 12th Previous Year Question Paper 2017 (CBSE)

Biology

SECTION – A 

(Q. Nos. 1 – 5 are of one marks each) 

Q.1. Name the type of cross that would help to find the genotype of a pea plant bearing violet flowers. 

Answer: Test cross 

 

Q.2. State two postulates of Oparin and Haldane with reference to the origin of life. 

Answer: (i) First form of life could have come from pre-existing non-living organic molecules / RNA & Protein

(ii) Formation of life was preceded by chemical evolution / formation of diverse organic molecules from inorganic constituents.

 

Q.2. Bt -toxins are released as inactive crystals in the bacterial body. What happens to it in

the cotton boll worm body that it kills the boll worm.(SET-III)

Ans: It is converted into an active protein (due to alkaline pH of the gut of the boll worm) , the toxin binds to midgut cells / create pores / causes cell swelling and lysis that kills the bollworm. 

 

Q.3. A herd of cattle is showing reduced fertility and productivity. Provide one reason and one suggestion to overcome this problem. 

Answer: Reason: Inbreeding depression / continuous inbreeding.

Suggestion: Should be mated with unrelated superior cattle of the same breed / out – breeding / out – crossing.

 

Q.3. Name the specific type of gene that is incorporated in a cotton plant to protect the plant against cotton bollworm infestation. (SET-II)

Answer: cry I Ac / cry II Ab

 

Q.4. What are Cry genes ? In which organisms are they present ? 

Ans. The genes which code for Bt toxin / Cry proteins / toxic proteins , Bacillus thuringiensis.

 

Q.5. An electrostatic precipitator in a thermal power plant is not able to generate high voltage of several thousands. Write the ecological implication because of it. 

Answer: Air Pollution // particulate matter / dust particles released in the air.

 

SECTION – B 

(Q Nos. 6-10 are of two marks each) 

Q.6. A pollen grain in angiosperm at the time of dehiscence from an anther could be 2-celled or 3-celled. Explain. How are the cells placed within the pollen grain when shed at a 2-celled stage ? 

Answer: • In 2-celled stage the mature pollen grain contains a generative and vegetative cell, whereas in 3- celled stage one vegetative cell and two male gametes are present.

• The generative cell floats in the cytoplasm of vegetative cell

 

Q.7. Differentiate between the genetic codes given below : 

(a) Unambiguous and Universal 

(b) Degenerate and Initiator 

Answer:  

(a) Unambiguous: One codon codes for only one amino acid. Universal: Genetic code/codons are (nearly) the same for all organisms from bacteria to humans.
(b)Degenerate: More than one codon coding for the same amino acid. Initiator: Start codon / AUG.

 

Q.8. Mention one application for each of the following : 

(a) Passive immunization 

(b) Antihistamine 

(c) Colostrum 

(d) Cytokinin-barrier 

Answer: (a) Provide preformed antibodies / antitoxins for quick response in case of infection by deadly microbes(tetanus) or snake bite.

(b) Reduces symptoms of allergy 

(c) Provides passive immunity / antibodies / Ig A to new born.

(d) Protection of non-infected cells from further viral infection.

 

Q.9. Name the microbes that help production of the following products commercially: 

(a) Statin 

(b) Citric acid 

(c) Penicillin 

(d) Butyric acid 

Answer: (a) Monascus purpureus 

(b) Aspergillus niger 

(c) Penicillium notatum 

(d) Clostridium butylicum 

 

Q.10. List four benefits to human life by eliminating the use of CFCs. 

Answer: (i) Delay in aging of skin 

(ii) Prevent damage to skin cells 

(iii) Prevent skin cancer 

(iv) Prevent snow blindness / inflammation of cornea 

(v) Prevent cataract (a) Unambiguous: 

(vi) Prevents ozone depletion 

(vii) Prevents global warming 

(viii) Reduces greenhouse effect 

(ix) Reduces odd climatic changes or El Nino effect 

OR 

Q.10. Suggest two practices giving one example of each, that help protect rare or threatened species. 

Ans: (1) In situ conservation , biodiversity hotspot / biosphere reserve / national parks /sanctuaries / Ramsar sites / sacred groves (Any one).

(2) Ex situ conservation , Zoological parks / botanical garden / wildlife safari parks / 

cryopreservation techniques / Tissue culture / seed bank / pollen banks.

 

Q.6. Name the type of immunity the colostrum provides to a newborn baby. Write giving an example where this type of immunity should be provided to a person.

Answer: Passive Immunity.

In case of infection by deadly microbes(tetanus) / snake bite where quick immune response is required =1

 

Q.8. Write the binomials of two fungi and mention the products/bioactive molecules they help to produce.

Answer: Trichoderma polysporum , cyclosporin A

Aspergillus niger,citric acid

Monascus purpureus , statin

Saccharomyces cerevisiae, ethanol / alcohol

Penicillium notatum , Penicillin

 

Q.7. Give the binomials of two types of yeast and the commercial bioactive products they help to produce.

Ans: Saccharomyces cerevisiae- ethanol / alcohol Monascus purpureus- statin 

 

Q.9. How many cells are present in the pollen grains at the time of their release from another ? Name the cells.

Ans. Pollen grain may be released at

        2-celled stage , one vegetative and one generative cell ,

        3-celled stage , one vegetative cell and two male gametes 

 

Q.10. Name the group of cells the HIV enters after getting into the human body. What happens in these cells and what are these cells subsequently referred to as ? Name the next group of cells the HIV attacks from here.

Ans. Macrophages , Reverse transcription , HIV Factory , helper T-lymphocytes (TH)

 

SECTION – C 

(Q Nos. 11-22 are of three marks each) 

Q.11. (a) Can a plant flowering in Mumbai be pollinated by pollen grains of the same species growing in New Delhi ? Provide explanations to your answer. 

(b) Draw the diagram of a pistil where pollination has successfully occurred. Label the  parts involved in reaching the male gametes to its desired destination. 

Answer: (a) Yes, By artificial means ( any relevant explanation). 

(b) Diagram with following labellings Stigma , Pollen tube , Synergid / Filiform Apparatus , Micropyle.

 

Q.12. Both Haemophilia and Thalassemia are blood related disorders in humans. Write their causes and the difference between the two. Name the category of genetic disorder they both come under. 

Answer: 

Haemophilia Thalassemia
Single protein involved in the clotting of blood is affected Defects in the synthesis of globin leading to formation of abnormal haemoglobin.
Sex linked reccessive disorder. Autosomal recessive disorder.
Blood does not clot. Results in anaemia.

 

Q.13. (a) List the two methodologies which were involved in human genome project. Mention how they were used. 

(b) Expand ‘YAC’ and mention what was it used for. 

Answer: (a) Expressed Sequence Tags , Identifying all the genes that are expressed as RNA Sequence Annotation , sequencing the whole set of genome coding or non coding sequences and later assigning different region with functions.

(b) Yeast Artificial Chromosome , used as cloning vectors (cloning / amplification ).

 

Q.14. Write the characteristics of Ramapithecus , Dryopithecus and Neanderthal man. 

Answer: Ramapithecus: hairy/ walked like gorillas and chimpanzees , more man like. 

Dryopithecus: hairy/ walked like gorillas and chimpanzees , more ape- like. 

Neanderthal man: brain size is 1400cc , used hides to protect their body / buried their dead.

 

Q.15. Name a human disease, its causal organism, symptoms (any three) and vector, spread by intake of water and food contaminated by human faecal- matter. 

Answer: Amoebiasis (Amoebic dysentery) , Entamoeba histolytica , constipation / abdominal pain / cramps / stools with excess mucus / blood clots (Any three symptoms) , Housefly.

Ascariasis, Ascaris , internal bleeding / muscular pain / fever / anaemia / blockage of intestinal passage (Any three symptoms), Housefly.

Typhoid, Salmonella typhi, high fever / weakness / stomach pain / constipation / headache / loss of appetite (Any three symptoms), Housefly.

OR 

Q.15. (a) Why is there a fear amongst the guardians that their adolescent wards may get 

trapped in drug/alcohol abuse ? 

(b) Explain ‘addiction’ and ‘dependence’ in respect of drug/alcohol abuse in youth. 

Defects in the synthesis of globin leading to formation of abnormal haemeoglobin Sex linked recessive disorder.

Autosomal recessive disorder  Blood does not clot Results in anaemia 

Answer: (a) Adolescents are easily affected by ( vulnerable to) peer pressure /adventure /curiosity / excitement / experimentation / media . 

(b) Addiction -Psychological attachment to certain effects such as Euphoria / temporary feeling of well-being. Dependence-Tendency of the body to show withdrawal syndrome / symptoms if regular doses of drug / alcohol is abruptly discontinued. 

 

Q.16. (a) Write the desirable characters a farmer looks for in his sugarcane crop. 

(b) How did plant breeding techniques help north Indian farmers to develop cane with 

desired characters ? 

Answer: (a) High yield , thick stem,high sugar content , ability to grow in their areas. 

(b) By crossing Saccharum officinarum / south Indian variety having desired characteristics with Saccharum barberi / north Indian low yield variety.

 

Q.17. Secondary treatment of the sewage is also called Biological treatment. Justify this statement and explain the process. 

Answer: Involves biological organism such as aerobic and anaerobic microbes / bacteria and fungi to digest / consume organic waste. 

Primary effluent is passed into aeration tank where vigorous growth of aerobic microbes (flocs) take place, BOD reduced (microbes consume major part of organic matter), effluent is passed to settling tank where flocs sediment to produce activated sludge , sludge is pumped to anaerobic sludge digester to digest bacteria and fungi.

 

Q.18. (a) Explain the significance of ‘palindromic nucleotide sequence’ in the formation of recombinant DNA. 

(b) Write the use of restriction endonuclease in the above process. 

Answer: (a) Palindromic nucleotide sequence is the recognition (specific) sequence present both on the vector and on a desired / alien DNA for the action of the same(specific) restriction endonuclease to act upon.

(b) Same restriction endonuclease binds to both the vector and the foreign DNA , cut each of the two strands of the double helix at specific points in their sugar phosphate backbone of recognition sequence for restriction endonucleases / palindromic sequence of vector and foreign DNA , to cut strand a little away from the centre of the palindrome sites, creates overhanging stretches /sticky ends . 

(b) If depict diagrammatically showing the above mentioned value points it can be accepted. 

 

Q.19. Describe the roles of heat, primers and the bacterium Thermus aquaticus in the process of PCR. 

Answer: Heat – Denaturation / separation of DNA into two strands.

Primer- Enzyme DNA Polymerase extend the primers using the nucleotides provided in the reaction and the genomic DNA as template. 

Thermus aquaticus – source of thermostable DNA polymerase / Taq polymerase. 

 

Q.20. Explain the various steps involved in the production of artificial insulin. 

Answer: Two DNA sequences corresponding to A and B polypeptide chains of human insulin were prepared , these were introduced into E.coli to produce A and B chains separately , these chains were extracted and combined by creating disulphide bonds.

 

Q.21. (a) “Organisms may be conformers or regulators.” Explain this statement and give one example of each. 

(b) Why are there more conformers than regulators in the animal world ? 

Answer: (a) Conformers- organisms which cannot maintain a constant internal environment under varying external environmental conditions / change body temperature and osmotic concentration with change in external environment eg. all plants / fishes / amphibians / reptiles.

Regulators – organisms which can maintain homeostasis (by physiological means or behavioural means ) // maintain constant body temperature and osmotic concentration eg. birds /mammals.

(b) Thermoregulation is energetically expensive for animals.

 

Q.22. Describe the inter-relationship between productivity, gross primary productivity and net productivity. 

Answer: Productivity is the rate of biomass production per unit area over a period of time , 

Gross primary productivity is the rate of production of organic matter during photosynthesis in an ecosystem , Net productivity is the gross primary productivity minus respiration losses ®.

 

Q.13. Explain the process of pollination in Vallisneria. How is it different in water-lily, which is also an aquatic plant ?

Answe: In Vallisneria pollination takes place through water , the female flower reach the surface of water by long stalk , male flowers / pollen grain released on to the surface of water , carried passively by water current reaching the female flowers / stigma.

In Water lily pollination takes place through wind or insect , female flower emerges above the surface of water and gets pollinated 

 

Q.15. What is disturbance in Hardy-Weinberg genetic equilibrium indicative of ? Explain how it is caused.

Answer: Disturbance in Hardy-Weinberg equilibrium is an indicator of change of frequency of alleles in a population , resulting in evolution.

It is caused by genetic drift / gene flow or gene migration / mutation / genetic recombination /natural selection. 

 

Q.18. Different animals respond to changes in their surroundings in different ways. Taking one example each, explain “some animals undergo aestivation while some others hiberna

tion”. How do fungi respond to adverse climatic conditions ?

Answer: Some animals go into aestivation to avoid summer related problems ( heat and desiccation) , eg. snails / fish ( any other suitable eg.) 

Some animals go into hibernation to avoid winter related problem ( extreme cold) eg. bear ( any other suitable eg.).

Fungi form thick walled spores and suspend their activities to respond to adverse climatic condition.

 

Q.11. Rearrange Ramapithecus, Australopithecus and Homo habilis in the order of thein evolution on the Earth. Comment on their evolutionary characteristics.

Answer: Ramapithecus Australopithecus Homo habilis.

Ramapithecus – hairy / walked like gorilla and chimpanzees / more man like.

Australopithecus – Hunted with stone weapons / ate fruit.

Homo habilis -Brain capacity 650- 800 cc / probably did not eat meat.

 

Q.16. (a) Trace the development of an endosperm after fertilisation with reference to coconut. Mention the importance of endosperm development.

(b) Write the importance of ‘pollen bank’.

Answer: (a) In coconut Primary Endosperm Nucleus (PEN-3n) undergoes successive nuclear divisions , give rise to free- nuclear endosperm known as coconut water , white kernel is the cellular endosperm , provides nourishment to the growing embryo.

(b) Storage / cryopreservation ( storage in liquid nitrogen at – 196 o C) , to use in crop breeding programmes.

 

Q.20. Describe the inter-relationship , between productivity, gross primary productivity and net productivity.

Answer: Productivity is the rate of biomass production per unit area over a period of time ,

Gross primary productivity is the rate of production of organic matter during photosynthesis in an ecosystem ,Net productivity is the gross primary productivity minus respiration losses ®.

 

Q.22. How do kangaroo rats and desert plants adapt themselves to survive in their extreme habitat ? Explain.

Answer:  Kangaroo rats- internal fat oxidation where water is a byproduct , excretes concentrated urine.

Desert Plants -thick cuticle / sunken stomata / leaves reduced to spines / deep roots /

Special photosynthetic pathway / CAM.

 

SECTION – D 

(Q No. 23 is of four mark) 

Q.23 . It is commonly observed that parents feel embarrassed to discuss freely with their adoles- cent children about sexuality and reproduction. The result of this parental inhibition is that the children go astray sometimes. 

(a) Explain the reasons that you feel are behind such embarrassment amongst some 

parents to freely discuss such issues with their growing children. 

(b) By taking one example of a local plant and animal, how would you help these parents 

to overcome such inhibitions about reproduction and sexuality ? 

Ans: (a) Illiteracy / conservative attitude / misconceptions / social myths / any other relevant point (Any two).

(b) If a student gives the clarity of the concept of reproduction and sexuality by taking any example of a plant and an animal with respect to reproductive organs, gamete formation, fertilization, sexual behaviour etc.

 

SECTION – E 

(Q Nos. 24-26 are of five marks each) 

Q.24. (a) When a seed of an orange is squeezed, many embryos, instead of one are observed. 

Explain how it is possible. 

(b) Are these embryos genetically similar or different ? Comment. 

Ans: (a) Polyembryony , nucellar cells surrounding embryo sac start dividing , protrude into the embryo 

sac and develop into many embryos. 

(b) These embryos are genetically similar, as produced from nucellar cells by mitotic division / formed without fertilisation (but different from the embryo formed by fertilization). 

OR 

Q.24. (a) Explain the following phases in the menstrual cycle of a human female: 

(i) Menstrual phase 

(ii) Follicular phase 

(iii) Luteal phase 

(b) A proper understanding of menstrual cycle can help immensely in family planning. 

Do you agree with the statement ? Provide reasons for your answer. 

Ans: (a) (i) Menstrual phase – first 3-5 days of the cycle where menstrual flow occurs due to break down of endometrial lining of the uterus, if the released ovum is not fertilised.

(ii) Follicular phase – from 5th to 14th day of the cycle where the primary follicles grow to become a fully mature Graafian follicle , endometrium of uterus regenerates , Graafian follicle ruptures to release ova (ovulation on 14th day). 

(iii) Luteal Phase – During 15th to 28th day remaining parts of graafian follicle transforms into the corpus luteum , secretion of progesterone (essential for maintenance of endometrium).

All these phases are under the influence of varying concentrations of pituitary and ovarian Hormone.

(b) Yes , can take appropriate precautions between 10th to 17th day of the menstrual cycle when the chances of fertilisation are high. 

 

Q.25. (a) Compare, giving reasons, the J-shaped and S-shaped models of population growth of a species.

               (b) Explain “fitness of a species” as mentioned by Darwin. 

Answer: 

J shaped – growth curve  S shaped- growth curve 
Resources are unlimited  Resources are limited 
Growth is exponential  Logistic Growth 
As resources are unlimited all individuals survive and reproduce  Fittest individual will survive and reproduce 
Growth Equation dN/dt=Rn (If explained) Growth Equation dN/dt=rN (k-N/K) (If explained)

Note – Marks to be awarded only if the corresponding difference is written. 

(b) When resources are limited , Competition occurs between individuals , the fittest will survive, 

who reproduce to leave more progeny. 

OR 

Q.25. (a) What is an ecological pyramid ? Compare the pyramids of energy, biomass and numbers. 

(b) Write any two limitations of ecological pyramids. 

Ans: (a) Graphical representation of the relationships among organisms at different trophic levels.

(b) It does not accommodate the food web / does not take into account the same species belonging to two or more trophic levels , Saprophytes are not given any place.

 

Q.26. (a) Describe the structure and function of a t-RNA molecule. Why is it referred to as an adapter molecule? 

(b) Explain the process of splicing of hn-RNA in a eukaryotic cell. 

Ans: (a) Clover-leaf shaped / inverted L shaped molecules has an anticodon loop with bases complementary to specific codon , has an amino acid acceptor end = 1+1 

As it reads the code on one hand and binds with the specific amino acid on the other hand.

(b) Introns are removed and exons are joined in a definite order. 

Process of splicing shown diagramatically. 

OR 

Q.26. Growth Equation dN/dt=rN (k-N/K) (If explained) Pyramid of Energy Pyramid of BioMass Pyramid of Numbers  Pyramid of Numbers Shows transfer of Energy from shows numbers of one trophic level to another 

organisms at each trophic level. Always upright Mostly upright but can be inverted Mostly upright can be 

inverted Shows transfer of amount of food/ biomass from one trophic level to another. Write the different components of a lac-operon in E.coli. Explain its expression while in an ‘open’ state. 

Ans: It consists of one regulatory gene(i) , promoter gene , operator gene , and three structural genes(z,y,a).

Lactose/ inducer binds to the repressor protein , makes it inactive so it cannot bind with operator, allows RNA Polymerase access to the promoter and transcription proceeds ,β -galactosidase , permease , transacetylase formed (by translation process for Lactose metabolism).

Q.25. (a) Explain Polygenic inheritance and Multiple allelism with the help of suitable examples.

(b) “Phenylketonuria is a good example that explains Pleiotropy.” Justify.

Answer: (a) Traits that are generally controlled by three or more genes , the phenotype reflects the contribution of each allele / effect of each allele is additive.

eg. Human skin colour , controlled by three genes (A , B, C).

In multiple allelism more than two alleles , govern the same character / phenotype.

eg . Human blood group (ABO system) , controlled by three different alleles (IA, IB, i). 

(b) In pleiotropy a single gene can exhibit multiple phenotypic expressions , in phenylketonuria single mutated gene express mental retardation and reduction in hair and skin pigmentation

OR

Q.25. (a) What is an operon ?

(b) Explain how a polycistronic structural gene is regulated by a common promoter and a

combination of regulatory genes in a lac-operon.

Answer: (a) An operon is a polycistronic structural gene which is regulated by a common promoter and regulator

gene / transcriptionally regulated system in which polycistronic structural gene is controlled by a common promoter and regulator gene.

(b) 

  • Lac operon consist of one regulatory gene i which codes for the repressor protein , promoter (P) and operator (o) are adjacent to gene i.
  • Structural genes z, y, a code for enzymes (â-galactosidase , permease and transacetylase respectively).
  • The regulator gene i synthesizes the repressor protein (all the time) , in the absence of inducer , the repressor protein binds to the operator region of the operon , prevents transcription (by RNA polymerase).
  • The repressor is inactivated in the presence of an inducer (lactose) that binds with it , this allows RNA polymerase access to promoter and transcription proceeds.

 

Q.24. (a) A pea plant bearing axial flowers is crossed with a pea plant bearing terminal flowers. The cross is carried out to find the genotype of the pea plant bearing axial flowers. Work out the cross to show the conclusions you arrive at.

(b) State the Mendel’s law of inheritance that is universally acceptable.

Answer: (i) If the plants is homozygous for the dominant trait

(ii) If the plants is heterozygous for the dominant trait A A a a (All plants with Axial Flower) A a a (50 % plants are with Axial ?ower and 50% plants with terminal ?ower) a

Conclusion : If all progeny show axial flowers ( dominant) the plant is homozygous (AA) ,

If 50 % of Progeny show Axial flower ( Dominant) and 50% Terminal flower ( Recessive) the plant is heterozygous.

(b) Law of Segregation , allelic pair segregate (separates) during gamete formation ( do not loose their identity ).

OR

Q.24. (a) Absence of lactose in the culture medium affects the expression of a Lac-operon in E. coli. Why and how ? Explain.

(b) Write any two ways in which the gene expression is regulated in eukaryotes.

Answer: (a) • Lactose acts as inducer thus absence of lactose switches off the operon.

• Repressor protein produced by regulatory gene ( i-gene ) is free ( in the absence of inducer ) ,

• Repressor protein binds with the operator gene ( o-gene ) ,

• Preventing RNA polymerase to transcribe the structural gene and operon is switched off.

If the above mentioned points are properly represented with help of schematic diagram.

(b) • Transcriptional level ( formation of primary transcripts )

• Processing level ( regulation of splicing )

• Transport of messengar RNA from nucleus to the cytoplasm

• Translational level 

 

Q.25. (a) When a seed of an orange is squeezed, many embryos, instead of one are observed. Explain how it is possible.

(b) Are these embryos genetically similar or different ? Comment.

Ans: (a) Polyembryony , nucellar cells surrounding embryosac start dividing , protrude into the embryosac and develop into many embryos = 1+ 1+ 1

(b) These embryos are genetically similar, as produced from nucellar cells by mitotic division / formed without fertilisation (but different from the embryo formed by fertilization)

OR

Q.25. (a) Explain the following phases in the menstrual cycle of” a human female:

(i) Menstrual phase

(ii) Follicular phase

(iii) Luteal phase

(b) A proper understanding of menstrual cycle can help immensely in family planning.

Do you agree with the statement ? Provide reasons for your answer.

Ans: (a) (i) Menstrual phase – first 3-5 days of the cycle where menstrual flow occurs due to break down of endometrial lining of uterus, if the released ovum is not fertilised.

(ii) Follicular phase – from 5th to 14th day of the cycle where the primary follicles grow to become a fully mature Graafian follicle , endometrium of uterus regenerates , Graafian follicle ruptures to release ova (ovulation on 14th day)

(iii) Luteal Phase – During 15th to 28th day remaining parts of graafian follicle transform into corpus luteum , secretion of progesterone (essential for maintenance of endometrium)

All these phases are under the influence of varying concentrations of pituitary and ovarian hormone.

(b) Yes , can take appropriate precautions between 10th to 17th day of the menstrual cycle when the chances of fertilisation are high.

Biology 12th Previous Year Question Paper 2018 (CBSE)

Biology

SECTION-A 

(Q. Nos. 1 – 5 are of one mark each) 

Q.1. Write the dual purpose served by Deoxyribonucleoside triphosphates in polymerisation. 

Ans. Acts as a substrate , provide energy (from the terminal two phosphates).

 

Q.2. Name two diseases whose spread can be controlled by the eradication of Aedes 

mosquitoes. 

Ans. Dengue , Chikungunya // Yellow Fever / Eastern Equine Encephalitis / West Nile Fever / Zika / Zika Virus Disease (Any two)..

 

Q.3. How do cytokine barriers provide innate immunity in humans ? 

Ans. Interferon (proteins) , secreted by virus infected cells (protect non – infected cells from further viral infection).

 

Q.4. Write the names of the following : 

(a) A 15 mya primate that was ape-like 

(b) A 2 mya primate that lived in East African grasslands 

Ans. (a) Dryopithecus. 

(b) Australopithecines / Australopithecus / Homo habilis. 

 

Q.5. Mention the chemical change that proinsulin undergoes, to be able to act as mature 

insulin. 

Ans. Removal of C – peptide (from proinsulin) 

 

SECTION-B 

(Q. Nos. 2 – 10 are of two marks each) 

Q.6. Your advice is sought to improve the nitrogen content of the soil to be used for the cultivation of a non-leguminous terrestrial crop. 

(a) Recommend two microbes that can enrich the soil with nitrogen. 

(b) Why do leguminous crops not require such enrichment of the soil ? 

Ans. (a) Azospirillum / Azotobacter / Anabaena / Nostoc / Oscillatoria / Frankia (Any 

two correct names of microbes)..

(If cyanobacteria mentioned. , but if along with cyanobacteria Anabaena / Nostoc / Oscillatoria mentioned then No mark on cyanobacteria) 

(b) They can fix atmospheric nitrogen , due to the presence of Rhizobium / N2 fixing bacteria 

in their root nodules..

 

Q.7. With the help of an algebraic equation, how did Hardy-Weinberg explain that in a given population the frequency of occurrence of alleles of a gene is supposed to remain the same through generations ? 

Ans. In a population of diploid organisms 

If the frequency of allele A = p and frequency of allele a = q. 

Expected genotype frequency under random mating are 

AA = p2 (for the AA homozygotes)

aa = q2 (for the aa homozygotes) 

Aa = 2pq (for the Aa heterozygotes). 

(In absence of selection , mutation , genetic drift or other forces allelic frequency p and q are constant through generations) 

Therefore p2 + 2pq +q2 = 1 = 1 

OR 

Q.7. Although a prokaryotic cell has no defined nucleus , yet DNA is not scattered throughout the cell. Explain. 

Ans. DNA is negatively charged , positively charged proteins , hold it in places , in large loops (in a region termed as nucleoid). × 4 

 

Q.8. How did a citizen group called Friends of the Arcata Marsh, Arcata, California, USA, help to improve water quality of the marshland using Integrated Waste Water Treatment ? Explain in four steps. 

Ans.- Water is treated by conventional method / sedimentation / filtration / chlorination 

– Water flows to six connected marshes 

– The water in marshes is seeded with appropriate plants / algae / fungi / 

bacteria 

– Which helps to neutralise the pollutants / assimilate the pollutants / absorb pollutants / Remove heavy metals. × 4 

 

Q.9. You have obtained a high yielding variety of tomatoes. Name and explain the procedure that ensures retention of the desired characteristics repeatedly in large populations of future generations of the tomato crop. 

Ans. – Tissue culture / micropropagation / somaclonal propagation / apomixis. 

– Explant / any part of plant taken out and grown (in a test tube / vessel) , 

– under sterile conditions , 

– in special nutrient medium (containing carbon source / sucrose , inorganic salt 

vitamins / amino acids and growth regulator). × 3 

 

Q.10. (a) Name the source plant of heroin drug. How is it obtained from the plant ? 

(b) Write the effects of heroin on the human body. 

Ans. (a) – Papaver somniferum / Poppy plant. 

– Extracted from latex of the plant / acetylation of morphine (obtained from the 

latex of plant). 

(b) Depressant , slows down body function..

 

SECTION-C 

(Q. Nos. 11 – 22 are of three marks each) 

Q.11. Draw a diagram of a mature human sperm. Label any three parts and write their 

functions. 

(Any three labelling).

Plasma membrane – Envelope of the sperm 

Acrosome – Filled with enzyme that help fertilization of ovum 

Mitochondria – Energy source for swimming 

Middle Piece – Possess mitochondria which is the energy source for swimming 

Tail – For movement of sperm 

Nucleus – Containing chromosomal material 

(Functions of the parts labelled ). × 3 

 

Q.12. (a) Expand VNTR and describe its role in DNA fingerprinting. 

(b) List any two applications of DNA fingerprinting technique. 

Ans. (a) VNTR – Variable Number of Tandem Repeat(s). 

– used as a probe (because of its high degree of polymorphism). 

(b) Forensic science / criminal investigation (any point related to forensic science) / determine population and genetic diversities / paternity testing / maternity testing / study of evolutionary biology (Any two).

 

Q.13. Differentiate between Parthenocarpy and Parthenogenesis. Give one example of each. 

Ans. 

Parthenocarpy Parthenogenesis 
– Formation of fruit without fertilization  – New organism develops without fertilization 
– e.g. banana / grapes / any other correct example. – e.g. Drones /male honey bee / turkey / rotifers / some lizards / any other correct example. 

 

Q.14. Medically it is advised to all young mothers that breastfeeding is the best for their 

newborn babies. Do you agree ? Give reasons in support of your answer. 

Ans.Yes, 

provides nutrition (calcium , fats , lactose ) / provides (passive) immunity / provides antibodies / Ig A. 

 

Q.15. Explain the mechanism of ‘sex determination’ in birds. How does it differ from that of 

human beings ? 

Ans. In birds ; 

Birds : female heterogamety / female produces (Z) type and (W) type of gametes. 

Humans : male heterogamety / male produces (X) and (Y) type of gametes. 

 

Q.16. (a) How has the development of bioreactor helped in biotechnology ? 

(b) Name the most commonly used bioreactor and describe its working. 

Ans. (a) Larger biomass / large volume of culture can be processed leading to higher yields of desired specific products (protein / enzymes) , under controlled conditions..

(b) Stirring type. 

– Mixing of reactor contents evenly (with agitator system or a stirrer). 

– Facilitates oxygen availability. 

– Temperature / pH / foam control // under optimum conditions. 

 

Q.17. Explain the roles of the following with the help of an example each in recombinant 

DNA technology: 

(a) Restriction Enzymes 

(b) Plasmids 

Ans. (a) It recognises a specific sequence of base pairs / palindromes, and cuts the DNA 

strand at a specific site..

eg. EcoRI / Hindiii or any other correct example. 

(b) Act as vectors / cloning of desired alien gene / foreign gene = 1 

eg. pBR322 / plasmid of Salmonella / plasmid of Agrobacterium / Ti Plasmid / Tumour inducing Plasmid. 

 

Q.18. Explain out-breeding, out-crossing and crossbreeding practices in animal husbandry. 

Out breeding – Breeding of unrelated animals (which may be between individual of 

same breed or between individuals of different species) = 1 

Outcrossing – (a kind of out breeding) Mating of animals within the same breed but having no common ancestors on either side of their pedigree upto 4 – 6 generations = 1 

Cross breeding – (another type of out breeding) Superior males of one breed are mated 

with superior females of another breed = 1 

 

Q.19. (a) Organic farmers prefer biological control of diseases and pests to the use of 

chemicals for the same purpose. Justify. 

(b) Give an example of a bacterium, a fungus and an insect that are used as 

biocontrol agents. 

Ans. (a) – Reduces dependence on toxic chemicals 

– Protects our ecosystem or environment 

– Protects and conserves non-target organisms / they are species – specific 

– These chemicals being non-biodegradable may pollute the environment 

permanently 

– These chemicals being non-biodegradable may cause biomagnification 

(b) Bacteria – Bacillus thuringiensis. 

Fungus – Trichoderma. 

Insect – Ladybird / Dragonfly / Moth or any other correct example. 

 

Q.20. (a) Differentiate between analogous and homologous structures. 

(b) Select and write analogous structures from the list given below : 

(i) Wings of butterflies and birds 

(ii) Vertebrate hearts 

(iii) Tendrils of bougainvillea and cucurbita 

(iv) Tubers of sweet potato and potato 

Ans. (a) Analogous – Anatomically not similar though perform similar functions / are a 

result of convergent evolution = 1 

Homologous – Anatomically similar (but perform different functions) / are a result 

of divergent evolution = 1 

(b) Option (i) Wings of butterflies and birds / (iv) Tubers of sweet potato and potato 

 

Q.21. (a) “India has greater ecosystem diversity than Norway.” Do you agree with the 

statement ? Give reasons in support of your answer. 

(b) Write the difference between genetic biodiversity and species biodiversity 

that exists at all levels of biological organisation. 

Ans. (a) Yes. 

India / tropical region Norway / temperate region 

– are less seasonal – more seasonal / 

/ more constant / more predictable / less constant / less predictable 

– promote niche specialisation – do not promote niche specialisation 

leading to greater biodiversity leading to low biodiversity 

– Species diversity increases as we – Species diversity decreases as we 

move towards equator move away from equator 

– More number of species exist – Less number of species exist 

(b) Genetic diversity – Diversity / variation within a species over its distributional range / same explained with the help of a correct example = 1 

Species diversity – Diversity / variation at a species level / same explained with 

the help of a correct example = 1 

OR 

Q.21. Explain the effect on the characteristics of a river when urban sewage is discharged into it. 

Ans. – Rise in organic matter , leads to increased microbial activity / growth of microbes.

– It results in a decrease in dissolved oxygen / rise in BOD / rise in Biochemical Oxygen 

Demand = 1 

– Leads to fish mortality / algal bloom / colour change / foul odour / increase in 

toxicity.

 

Q.22. How has the use of Agrobacterium as vectors helped in controlling Meloidogyne 

incognitia infestation in tobacco plants ? Explain in correct sequence. 

Ans. – Using Agrobacterium vector nematode specific genes introduced into host plant 

– Sense and antisense strands of mRNA are produced 

– ds RNA is formed 

– ds RNA initiates RNAi 

– Prevents translation of mRNA / silencing of mRNA of parasite / nematode 

– Parasite will not survive 

 

SECTION-D 

(Q. Nos. 23 is of four marks) 

Q.23. Looking at the deteriorating air quality because of air pollution in many cities of the country, the citizens are very much worried and concerned about their health. The doctors have declared health emergency in the cities where the air quality is very severely poor. 

(a) Mention any two major causes of air pollution. 

(b) Write any two harmful effects of air pollution to plants and humans. 

(c) As a captain of your school Eco-club, suggest any two programmes you would plan to organise in the school so as to bring awareness among the students on how to check air pollution in and around the school. 

Ans. (a) Vehicular discharge / smoke from industries / burning of agricultural wastes / smoke from incinerator / dust / smoke from thermal plants or any other correct cause 

(b) Reduces growth of plants / reduces yields of crops / premature death of plants / respiratory problems / acid rain / any other relevant point (Any two – one from plant and one from human)

(c) Plantation drive / awareness programmes through posters / nukkad natak / film show 

/ rallies / debates or any other 

 

SECTION-E 

(Q. Nos. 24 – 26 are of five marks each) 

Q.24. (a) Describe any two devices in a flowering plant which prevent both autogamy and 

geitonogamy. 

(b) Explain the events upto double fertilisation after the pollen tube enters one of 

the synergids in an ovule of an angiosperm. 

Ans. (a) – Dioecy / production of unisexual flowers (in different plants) 

– Self incompatibility. 

(b) – Pollen tube releases 2 male gametes in the cytoplasm of synergid 

– One male gamete fuses with egg cell / syngamy , resulting in diploid zygote 

– Other male gamete fuses with polar nuclei / triple fusion , to form triploid PEN (Primary Endosperm Nucleus) / PEC (Primary Endosperm Cell). 

OR 

Q.24. (a) Explain menstrual cycle in human females. 

(b) How can the scientific understanding of the menstrual cycle of human females 

help as a contraceptive measure ? 

Ans. (a) – Menstrual Phase – Menstrual flow occurs / due to breakdown of endometrial 

lining of uterus , when fertilization does not occur 

– Follicular Phase – Primary follicles grow into mature graafian follicles and endometrium regenerates through proliferation , changes induced by pituitary and ovarian hormones 

– Ovulatory Phase – LH surge , induces rupture of graafian follicle and release 

of secondary oocyte / ovum during middle of cycle (i.e. 14th day) 

– Luteal phase – Ruptured graafian follicle transforms into the corpus luteum which secrete large amounts of progesterone , essential for maintaining endometrium.

(b) Because ovulation occurs during mid cycle chances of fertilisation are very high 

so , couples should abstain from coitus between day 10 – 17.

 

Q.25. (a) Write the scientific name of the organism Thomas Hunt Morgan and his colleagues worked with for their experiments. Explain the correlation between linkage and recombination with respect to genes as studied by them. 

(b) How did Sturtevant explain gene mapping while working with Morgan ? 

Ans. (a) Drosophila melanogaster. 

They observed that two genes (located closely on a chromosome) did not segregate independently of each other (F2 ratio deviated significantly from 9 : 3 : 3 : 1). 

Tightly linked genes tend to show fewer (less) recombinant frequency of parental traits / show higher (more) frequency of parental type. 

Loosely linked genes show higher percentage (more) of recombinant frequency of parental traits / lower frequency percentage of parental type. 

Genes present on the same chromosome are said to be linked and the recombinant frequency depends on their relative distance on the chromosome. 

(b) He used the frequency of recombination between gene pairs on the same chromosome , as a measure of the distance between genes and mapped their position on the chromosome 

OR 

 

Q.25. (a) State the ‘Central dogma’ as proposed by Francis Crick. Are there any 

exceptions to it ? Support your answer with a reason and an example. 

(b) Explain how the biochemical characterisation (nature) of ‘Transforming Principle’ was determined, which was not defined from Griffith’s experiments. 

Ans. (a)

Yes, in some viruses flow of information is in reverse direction/reverse transcription.

(b) Protein and DNA and RNA were purified from heat killed S strain / smooth 

Streptococcus / Diplococcus pneumoniae. 

Protein + Protease → transformation occurred (R cell to S type). 

RNA + RNA base → transformation occurred (R cell to S type). 

DNA + DNAse → transformation inhibited. 

Hence DNA alone is the transforming material. 

 

Q.26. (a) Following are the responses of different animals to various abiotic factors. 

Describe each one with the help of an example. 

(i) Regulate 

(ii) Conform 

(iii) Migrate 

(iv) Suspend 

(b) If 8 individuals in a population of 80 butterflies die in a week, calculate the death rate of population of butterflies during that period. 

Ans.

(a) (i) Regulate – Maintain constant internal temperature / osmotic concentration /homeostasis. e.g. birds / mammals. 
(ii) Conform – Do not maintain constant internal temperature / osmotic concentration / No homeostasis. e.g. any one example of animals other than birds and mammals. 
(iii) Migrate – Temporary movement of organisms from the stressful of habitats to hospitable areas and return when stressful period is over. e.g. birds from Siberia / or any other correct example. 
(iv) Suspend – Reducing / minimising the metabolic activities during unfavourable conditions. e.g. Polar bear / amphibian / snails / fish / any other examples of animals. 

(b) Death rate = 8/80 = 0.1, individuals per butterfly per week.

OR 

 

Q.26. (a) What is a trophic level in an ecosystem ? What is ‘standing crop’ with reference to it ? 

(b) Explain the role of the ‘first trophic level’ in an ecosystem. 

(c) How is the detritus food chain connected with the grazing food chain in a 

natural ecosystem ? 

Ans. (a) Specific place of an organism in a food chain , mass of living material (biomass) at each trophic level at a particular time.

(b) First trophic level has producers / autotrophs , which trap solar energy / to produce 

food (photosynthesis). 

(c) Organisms of the Detritus food chain (DFC) are the prey to the Grazing food chain (GFC) organism , the dead remains of GFC are decomposed into simple inorganic materials which are absorbed by DFC organisms. 

Maths 10th Previous Year Question Paper 2018 (CBSE)

Maths

Section – A

Q.1. If x = 3 is one root of the quadratic equation x2 – 2kx – 6 = 0, then find the value of k. 

Solution:

Given quadratic equation is, x2 – 2kx – 6 = 0

x = 3 is a root of above equation, then

(3)2 – 2k (3) – 6 = 0

 ⇒ 9 – 6k – 6 = 0

⇒ 3 – 6k = 0

 ⇒ 3 = 6k

⇒ k = 3/6

⇒ k =½

 

Q.2. What is the HCF of the smallest prime number and the smallest composite number? 

Solution:

Smallest prime number = 2

Smallest composite number = 4

Prime factorisation of 2 is 1 × 2

Prime factorisation of 4 is 1 × 22

HCF (2, 4) = 2

 

Q.3.Find the distance of a point P(x, y) from the origin. 

Solution:

The given point is P (x, y).

The origin is O (0, 0)

The distance of point P from the origin,

 

Q.4. In an AP if the common difference (d) = -4 and the seventh term (a7) is 4, then find the first term. 

Solution:

Given,

d = -4, a7 = 4

a + 6d = 4

⇒ a + 6(-4) = 4

⇒ a – 24 = 4

⇒ a = 4 + 24

⇒ a = 28

 

Q.5. What is the value of (cos2 67° – sin2 23°) ? 

Solution:

We have, cos2 67° – sin2 23°

= cos2 67° – cos2 (90° – 23°)          [∵ sin (90° – θ) = cos θ]

= cos2 67° – cos2 67°

= 0

 

Q.6. Given ΔABC ~ ΔPQR, if AB/PQ=1/3  then find arΔABC/arΔPQR

Solution:

 

Section – B

Q.7.Given that √2 is irrational, prove that (5 + 3√2) is an irrational number. 

Solution:

Given, √2 is an irrational number.

Let √2 = m

Suppose, 5 + 3√2 is a rational number.

 But a-5b/3b is rational number, so m is rational number which contradicts the fact that m = √2 is irrational number.

So, our supposition is wrong.

Hence, 5 + 3√2 is also irrational.

Hence Proved.

Q.8.In fig. 1, ABCD is a rectangle. Find the values of x and y.

Solution:

Given, ABCD is a rectangle.

AB = CD

⇒ 30 = x + y

or 

⇒ x + y = 30 …(i)

Similarly, AD = BC

⇒ 14 = x – y

or 

⇒ x – y = 14 …(ii)

On adding eq. (i) and (ii), we get

2x = 44

⇒ x = 22

Putting the value of x in eq. (i), we get

 22 + y = 30

 ⇒ y = 30 – 22

 ⇒ y = 8

 So, x = 22, y = 8.

 

Q.9.Find the sum of the first 8 multiples of 3. 

Solution:First 8 multiples of 3 are 3, 6, 9,….. up to 8 terms

 We can observe that the above series is an AP with

 a = 3, d = 6 – 3 = 3, n = 8

 Sum of n terms of an A.P is given by,

 

Q.10.Find the ratio in which P(4, m) divides the line segment joining the points A(2, 3) and B(6, -3). Hence find m. 

Solution:Let P divides line segment AB in the ratio k : 1

 Coordinates of P

 

Q.11. Two different dice are tossed together. Find the probability:

 (i) of getting a doublet.

 (ii) of getting a sum 10, of the numbers on the two dice. 

Solution:

 Total outcomes on tossing two different dice = 36

 (i) A: getting a doublet

 A = {(1, 1), (2, 2), (3, 3), (4, 4), (5, 5), (6, 6)}

 Number of favourable outcomes of A = 6

 (ii) B: getting a sum 10.

 B = {(4, 6), (5, 5), (6, 4)}

 Number of favourable outcomes of B = 3

 

Q.12. An integer is chosen at random between 1 and 100. Find the probability that it is:

 (i) divisible by 8.

 (ii) not divisible by 8. 

Solution: The total number are 2, 3, 4, …….. 99

 (i) Let E be the event of getting a number divisible by 8.

 E = {8, 16, 24, 32, 40, 48, 56, 64, 72, 80, 88, 96} = 12

 (ii) Let E’ be the event of getting a number not divisible by 8.

Then, P(E’) = 1 – P(E) = 1 – 0.1224 = 0.8756

 

Section – C

Q.13. Find HCF and LCM of 404 and 96 and verify that HCF × LCM = Product of the two given numbers. 

Solution:

 Prime factorization of 404 = 2 × 2 × 101

 Prime factorization of 96 = 2 × 2 × 2 × 2 × 2 × 3

 HCF = 2 × 2 = 4

 And LCM = 2 × 2 × 2 × 2 × 2 × 3 × 101 = 9696

 HCF = 4, LCM = 9696

 Verification:

 HCF × LCM = Product of the two given numbers

 4 × 9696 = 404 × 96

 38784 = 38784

 Hence Verified.

 

Q.14. Find all zeroes of the polynomial (2x4 – 9x3 + 5x2 + 3x – 1) if two of its zeroes are (2 + √3) and (2 – √3). 

Solution:

Here, p(x) = 2x4 – 9x3 + 5x2 + 3x – 1

And two of its zeroes are (2 + √3) and (2 – √3).

Quadratic polynomial with zeroes is given by,

 {x – (2 + √3)}. {x – (2 – √3)}

 ⇒ (x – 2 – √3) (x – 2 + √3)

 ⇒ (x – 2)2 – (√3)2

 ⇒ x2 – 4x + 4 – 3

 ⇒ x2 – 4x + 1 = g(x) (say)

 Now, g(x) will be a factor of p(x) so g(x) will be divisible by p(x)

 For other zeroes,

 2x2 – x – 1 = 0

 2x2 – 2x + x – 1 = 0

 or 

2x (x – 1) + 1 (x – 1) = 0

(x – 1) (2a + 1) = 0

x – 1 = 0 and 2x + 1 = 0

x = 1, x = -½

Zeroes of p(x) are

 1, -½, 2 + √3 and 2 – √3.

 

Q.15. If A(-2, 1) and B(a, 0), C(4, b) and D( 1, 2) are the vertices of a parallelogram ABCD, find the values of a and b. Hence find the lengths of its sides. 

 OR

Q.15. If A(-5, 7), B(-4, -5), C(-1, -6) and D(4, 5) are the vertices of a quadrilateral, find the area of the quadrilateral ABCD.

Solution:

Given ABCD is a parallelogram.

 

Q.16. A plane left 30 minutes late than its scheduled time and in order to reach the destination 1500 km away in time, it had to increase its speed by 100 km/h from the usual speed. Find its usual speed. 

Solution: Let the usual speed of plane be x km/h.

 Increased speed = (x + 100) km/h.

 Distance to cover = 1500 km.

 Time taken by plane with usual speed = 1500 /x hr

 Time taken by plane with increased speed = 1500 /100 + x

 According to the question,

 x2 + 100x = 300000

 x2 + 100x – 300000 = 0

 x2 + 600x – 500x – 300000 = 0

 x(x + 600) – 500(x + 600) = 0

 (x + 600) (x – 500) = 0

 Either x + 600 = 0 ⇒ x = -600 (Rejected)

 or 

 x – 500 = 0 ⇒ x = 500

 Usual speed of plane = 500 km/hr.

 

Q.17. Prove that the area of an equilateral triangle described on one side of the square is equal to half the area of the equilateral triangle described on one of its diagonal. 

 OR

Q.17. If the area of two similar triangles is equal, prove that they are congruent.

Solution: Let ABCD be a square with side ‘a’.

 

Q.18. Prove that the lengths of tangents drawn from an external point of a circle are equal. 

Solution:

Given: A circle with centre O on which two tangents PM and PN are drawn from an external point P.

 To Prove: PM = PN

 Construction: Join OM, ON and OP

 Proof: Since tangent and radius are perpendicular at point of contact,

 ∠OMP = ∠ONP = 90°

 In ΔPOM and ΔPON,

 OM = ON (Radii)

 ∠OMP = ∠ONP

 PO = OP (Common)

 ΔOMP = ΔONP (RHS cong.)

 PM = PN (C.P.C.T)

 Hence Proved.

 

Q.19. If 4 tanθ = 3, evaluate [(4sinθ – cosθ + 1)/ (4sinθ + cosθ – 1)]

 OR

Q.19. If tan 2A = cot (A – 18°), where 2A is an acute angle, find the value of A.

Solution:

 Given, 4 tan θ = 3

 ⇒ tan θ = ¾ (=P/B)

 OR

 Solution: Given, tan 2A = cot (A – 18°)

 ⇒ cot (90° – 2A) = cot (A – 18°)

 [∵ tan θ = cot (90° – θ)]

 ⇒ 90° – 2A = A – 18°

 ⇒ 90° + 18° = A + 2A

 ⇒ 108° = 3A

 ⇒ A = 36°

 

Q.20. Find the area of the shaded region in Fig. 2, where arcs are drawn with centres A, B, C and D intersect in pairs at mid-points P, Q, R and S of the sides AB, BC, CD and DA respectively of a square ABCD of side 12 cm. [Use π = 3.14] 

 Solution:

 Given, ABCD is a square of side 12 cm.

 P, Q, R and S are the midpoints of sides AB, BC, CD and AD respectively.

 Area of shaded region = Area of square – 4 × Area of quadrant

 = a2 – 4 × ¼πr2 

 = (12)2 – 3.14 × (6)2

 = 144 – 3.14 × 36

 = 144 – 113.04

 = 30.96 cm2

 

Q.21. A wooden article was made by scooping out a hemisphere form each end of a solid cylinder, as shown in Fig. 3. If the height of the cylinder is 10 cm and its base is of radius 3.5 cm. Find the total surface area of the article. 

 OR

Q.21. A heap of rice is in the form of a cone of base diameter 24 m and height 3.5 m. Find the volume of the rice. How much canvas cloth is required to just cover the heap?

Solution:

 Given, Radius (r) of cylinder = Radius of hemisphere = 3.5 cm.

 Total SA of article = CSA of cylinder + 2 × CSA of hemisphere

 Height of cylinder, h = 10 cm

 TSA = 2πrh + 2 × 2πr2

 = 2πrh + 4πr2

 = 2πrh (h + 2r)

 = 2 × ²²⁄7× 3.5 (10 + 2 × 3.5)

 = 2 × 22 × 0.5 × (10 + 7)

 = 2 × 11 × 17

 = 374 cm2

 OR

 Solution: Base diameter of cone = 24 m.

 Radius r = 12 m

 Height of cone, h = 3.5 m

 Volume of rice in conical heap = ⅓πr2h

 = ⅓ × ²²⁄7 × 12 × 12 × 3.5 = 528 cm3

 

Q.22. The table below shows the salaries of 280 persons:

Calculate the median salary of the data.

Solution:

 N2= 2802  = 140

The cumulative frequency just greater than 140 is 182.

Median class is 10 -15.

 l = 10, h = 5, N = 280, c.f. = 49 and f = 133

 

Section – D

Q.23. A motorboat whose speed is 18 km/hr in still water takes 1 hr more to go 24 km upstream than to return downstream to the same spot. Find the speed of the stream. 

 OR

Q.23. A train travels at a certain average speed for a distance of 63 km and then travels at a distance of 72 km at an average speed of 6 km/hr more than its original speed. If it takes 3 hours to complete the total journey, what is the original average speed?

Solution:

Given, speed of motorboat instil

 water = 18 km/hr.

 Let speed of stream = x km/hr.

 Speed of boat downstream = (18 + x) km/hr.

 And speed of boat upstream = (18 – x) km/hr.

 Time of the upstream journey = 2418-x 

 Time of the downstream journey = 2418+x

 According to the question,

 ⇒ x2 + 48x – 324 = 0

 ⇒ x2 + 54x – 6x – 324 = 0

 ⇒ x(x + 54) – 6(x + 54) = 0

 ⇒ (x + 54)(x – 6) = 0

 Either x + 54 = 0 ⇒ x = -54

 Rejected, as speed cannot be negative

 Or

 x – 6 = 0 ⇒ x = 6

 Thus, the speed of the stream is 6 km/hr.

 OR

 Solution: Let the original average speed of train be x km/hr.

 Increased speed of train = (x + 6) km/hr.

 Time taken to cover 63 km with average speed = 64x hr.

 Time taken to cover 72 km with increased speed = 72x+6 hr.

 According to the question,

⇒ 135x + 378 = 3(x2 + 6x)

 ⇒ 135x + 378 = 3x2 + 18x

 ⇒ 3x2 + 18x – 135x – 378 = 0

 ⇒ 3x2 – 117x – 378 = 0

 ⇒ 3(x2 – 39x – 126) = 0

 ⇒ x2 – 39x – 126 = 0

 ⇒ x2 – 42x + 3x – 126 – 0

 ⇒ x(x – 42) + 3(x – 42) = 0

 ⇒ (x – 42) (x + 3) = 0

 Either x – 42 = 0 ⇒ x = 42

 or 

x + 3 = 0 ⇒ x = -3

 Rejected (as speed cannot be negative)

 Thus, average speed of train is 42 km/hr.

 

Q.24. The sum of four consecutive numbers in an AP is 32 and the ratio of the product of the first and the last term to the product of two middle terms is 7:15. Find the numbers. 

Solution: Let the first term of AP be a and d be a common difference.

 Let your consecutive term of an AP be a – 3d, a – d, a + d and a + 3d

 According to the question,

 a – 3d + a – d + a + d + a + 3d = 32

 ⇒ 4a = 32

 ⇒ a = 8 …(i)

 Also,

 (a – 3d) (a + 3d) : (a – d) (a + d) = 7 : 15

 For d = 2, four terms of AP are,

 a – 3d = 8 – 3 (2) = 2

 a – d = 8 – 2 = 6

 a + d = 8 + 2 = 10

 a + 3d = 8 + 3(2) = 14

 For d = -2, four term are

 a – 3d = 8 – 3(-2) = 14

 a – d = 8 – (-2) = 10

 a + d = 8 + (-2) = 6

 a + 3d = 8 + 3 (-2) = 2

 Thus, the four terms of AP series are 2, 6, 10, 14 or 14, 10, 6, 2.

 

Q.25. In an equilateral ∆ABC, D is a point on side BC such that BD = ⅓ BC. Prove that 9(AD)2 = 7(AB)2

OR

Q.25. Prove that, in a right triangle, the square on the hypotenuse is equal to the sum of the squares on the other two sides.

Solution:

 Given, ABC is an equilateral triangle and D is a point on BC such that BD = ⅓  BC.

To prove: 9AD2 = 7AB2

 Construction : Draw AE ⊥ BC

 Proof: BD = ⅓ BC …(i) (Given)

 AE ⊥ BC

 We know that perpendicular from a vertex of equilateral triangle to the base divides base in two equal parts.

 BE = EC = ½ BC …(ii)

 In ∆AEB,

 AD2 = AE2 + DE2 (Pythagoras theorem)

 or 

AE2 = AD2 – DE2 …(iii)

 Similarly, In ∆AEB,

 AB2 = AE2 + BE2

 OR

Solution: Given: ∆ABC is a right angle triangle, right-angled at A.

 To prove : BC2 = AB2 + AC2

 Construction : Draw AD ⊥ BC.

 Proof: In ∆ADB and ∆BAC,

 ∠B = ∠B (Common)

 ∠ADB = ∠BAC (Each 90°)

 ∆ADB ~ ∆BAC (By AA similarity axiom)

 ABBC= BDAB (CPCT)

 AB2 = BC × BD

 Similarly,

 ∆ADC ~ ∆CAB

 ACBC= DCAC

 AC2 = BC × DC …(ii)

 On adding equation (i) and (ii)

 AB2 + AC2 = BC × BD + BC × CD = BC (BD + CD) = BC × BC

 AB2 + AC2 = BC2

 BC2 = AB2 + AC2

Hence Proved.

 

Q.26. Draw a triangle ABC with BC = 6 cm, AB = 5 cm and ∠ABC = 60°. Then construct a triangle whose sides are ¾ of the corresponding sides of the ∆ABC. 

Solution:

Draw a line segment BC = 6 cm.

Construct ∠XBC = 60°.

With B as centre and radius equal to 5 cm, draw an arc intersecting XB at A.

Join AC. Thus, ∆ABC is obtained.

Draw an acute angle ∠CBY below of B.

Mark 4-equal parts on BY as B1, B2, B3 and B4

Join B4 to C.

From By draw a line parallel to B4C intersecting BC at C’.

Draw another line parallel to CA from C’, intersecting AB at A’.

∆A’BC’ is required triangle which is similar to ∆ABC such that BC’ = ¾  BC.

 

Q.27.

 Solution:

 

Q.28. The diameters of the lower and upper ends of a bucket in the form of a frustum of a cone are 10 cm and 30 cm respectively. If its height is 24 cm, find:

 (i) The area of the metal sheet used to make the bucket.

 (ii) Why we should avoid the bucket made by ordinary plastic? [Use π = 3.14] 

Solution: Given, Height of frustum, h = 24 cm.

 Diameter of lower end = 10 cm

 Radius of lower end, r = 5 cm.

 Diameter of upper end = 30 cm

 Radius of upper end, R = 15 cm.

(i) Area of metal sheet used to make the bucket = CSA of frustum + Area of base

 = πl(R + r) + πr2

 = π[26 (15 + 5) + (5)2]

 = 3.14 (26 × 20 + 25)

 = 3.14 (520 + 25)

 = 3.14 × 545

 = 1711.3 cm2

(ii) We should avoid the bucket made by ordinary plastic because plastic is harmful to the environment and to protect the environment its use should be avoided.

 

Q.29. As observed from the top of a 100 m high lighthouse from the sea-level, the angles of depression of two ships are 30° and 45°. If one ship is exactly behind the other on the same side of the lighthouse, find the distance between the two ships. [Use √3 = 1.732] 

Solution: Let AB be the lighthouse and two ships are at C and D.

 Distance between two ships = y – x

 = 100√3 – 100 [from equation (i) and (ii)]

 = 100 (√3 – 1)

 = 100(1.732 – 1)

 = 100 (0.732)

 = 73.2 m

 

Q.30. The mean of the following distribution is 18. Find the frequency f of the class 19-21. 

 OR

Q.30. The following distribution gave the daily income of 50 workers of a factory:

Convert the distribution above to a less than type cumulative frequency distribution and draw its give.

Solution:

Social Science 10th Previous Year Question Paper 2019 SET-I (CBSE)

Social Science

Section – A

Q.1. Explain the aim to form ‘Zollverein’ a Customs Union, in 1834 in Germany. 

OR

Q.1. Explain the main reason responsible for the eruption of a major protest in Saigon Native Girls School in Vietnam in 1926. 

Answer: Zollverein was a customs union formed in 1834 at the initiative of Prussia. The union abolished tariff barriers and internal custom dues and was willing to establish free trade with neighbouring states. It reduced the number of currencies from thirty to two. Most German states joined the Zollverein.

OR

Answer: A major protest occurred in Saigon Native Girls School in 1926. This protest erupted because a Vietnamese girl sitting in one of the front seats was asked to move back and make space for a French student. She was expelled when she refused to obey the order of the Principal. Other students supporting her were also expelled by the principal who was a colon. This led to spread of open protests.

 

Q.2. Why was printing of textbooks sponsored by the Imperial State in China? 

OR

Q.2. Why did Chandu Menon give up the idea of translation of ‘English Novels’ in Malayalam?

Answer: The printing of textbooks were sponsored by the Imperial State in China because China possessed a large bureaucratic system, which recruited their personnel through Civil service examinations. That is why, textbooks were printed in large numbers to provide them study material.

OR

Answer: The readers in Kerala were not familiar with the lifestyle of the characters portrayed in english Novels. Direct translation of english novels into Malayalam was thus making the novels boring because the readers could not relate. Thus, Chandu menon gave up the idea of translating them and wrote a story in Malayalam instead.

 

Q.3. How has Shillong solved the problem of acute shortage of water? 

OR

Q.3. How has Tamil Nadu solved the problem of acute shortage of water ?

Answer: Shillong has been able to deal with the problem of acute shortage of water by setting up Bamboo drip irrigation systems.

OR

Answer: Tamil Nadu has been able to deal with the problem of acute shortage of water by adopting Rooftop water harvesting techniques.

 

Q.4. How did the feeling of alienation develop among the Sri Lankan Tamils?

Answer: The measures of the act of 1956 introduced by Sinhalese Government made the Sri Lankan Tamils feel alienated. They felt that none of the major political parties led by the Buddhist Sinhalese were sensitive towards their language and culture. They also felt that the constitution and policies of the government denied them equal political rights, discriminated against them in terms of jobs and other opportunities by ignoring their interests.

 

Q.5. What may be a developmental goal of farmers who depend only on rain for growing crops?

OR

Q.5. What may be a developmental goal of urban unemployed youth?

Answer: The developmental goal for a farmer who depends only on rain for irrigating his crops might be to have access to better water harvesting and irrigation techniques.

OR

Answer: The developmental goal for an urban unemployed youth would be to get a decent job suitable to his/her qualifications and skills.

 

Q.6. Give one example each of modem currency and older currency. 

Answer: An example of modem currency is the plastic money that we use in the form of debit and credit cards.

An example of older currency is the bronze coins that were used in earlier times.

 

Q.7. If you want to purchase an electrical valuable good, what logo would you like to see to confirm its quality?

 

Section -B

Q.8. Describe the great economic hardship that prevailed in Europe during the 1930s. 

OR

Q.8. Describe the serious problem faced by the modem part of Hanoi in 1903.

Answer: Great economic hardships were faced by the people of Europe in the 1930s. Some of the difficulties that they faced are:

  • The ratio of the rise of the population was larger than that of employment generation. People from rural areas were migrating to cities in search of employment, which was not as easily available because of overcrowding.
  • Small producers in towns (especially textile-producing industries) were often overthrown by the cheap machines. They faced stiff competition from imports from England.
  • Peasants still suffered under the burden of feudal dues and obligations in some regions of Europe. Rise of food prices and unemployment led to widespread pauperism in the country.

OR

Answer: The French sought to create a modem Vietnam and decided to rebuild Hanoi. The latest ideas about architecture and modem engineering skills were employed to build a new and ‘modem’ city.

(i) However, in 1903, the modem part of Hanoi was struck by bubonic plague.

(ii) The French part had wide avenues and well-planned sewer system, while the native quarter was not provided with any modem facilities. Thus, the refuse from the old city drained straight out into the river or, during heavy rains of floods, overflowed into the streets which become the cause of the plague.

(iii) Large sewers became a breeding ground for rats which began to enter the well-cared-for homes of the French through the sewage pipes, followed by massive rat hunt programme. This deteriorated the conditions. The whole city was under severe influence of this plague.

 

Q.9. How had the printing press created a new culture of reading in Europe? Explain with examples. 

OR

Q.9. How did Charles Dickens depicted the terrible effects of industrialisation on people’s lives and characters? Explain with examples.

Answer: With the introduction of the Printing Press, a new wave of print culture began in Europe. It was defined by accelerated production of books and printed material. The mass production of books lead to decrease in the prices of the books and their circulation increased. The reading culture was not restricted only to the elites but now, even the common people began to have easy access to these books.

Printers also focused on publishing the folk tales and ballads, well illustrated with pictures so that the books could be enjoyed even by a less educated audience of the villages. The books gave an opportunity to more and more people to come in contact with the ideas of philosophers and leading thinkers of the time. Thus, this changed the reading culture of Europe widely.

OR

Answer: Hard times was the tenth novel written by Charles Dickens. It was published in 1854. The process of industrialization and its effect on the labour force was the main theme of this novel. Set in the backdrop of a fictitious industrial city of Coketown, the novel sketches the condition of the then cities that were full of machinery, chimneys and smoke. The labours of the industries were considered to be ‘hands’ of the industries.

The economic pressures of the time had reduced human beings to mere instruments of production. One of his other novels, Oliver Twist also revolves around the same theme of consequences of industrialisation. Thus, Dickens proved that the prevailing idea of Utilitarianism which believed in the “the greatest amount of happiness for the greatest number of people” actually lead to the misery of several other or the happiness of the influential lead to the misery of the labourers.

 

Q.10. Describe any three main features of ‘Alluvial soil’ found in India.

OR

Q.10. Describe any three main features of ‘Black soil’ found in India.

Answer: Major characteristics of Alluvial Soil are:

  • Alluvial soil is considered as one of the most fertile soils. Alluvial soil covers the entire northern plains in India.
  • Alluvial soil contains sand, silt and clay mainly due to silt deposited by Indo-Gangetic-Brahmaputra rivers.

According to age, it is classified into Bangar (old alluvial) and Khadar (new alluvial).

  • Alluvial soil contains an ample amount of potash, phosphoric acid and lime. This soil is ideal for the growth of crops like sugarcane, wheat and rice etc.

OR

Answer: Major characteristics of Black soil are:

  • Black soil is fine textured and clayey in nature.
  • Black soil has high amount of lime, iron, magnesium and generally low quantities of ‘Phosphorus, Nitrogen and organic matter.
  • It is formed from weathered lava rocks, thus is black in colour.
  • It has a high clay content and therefore is highly retentive of water. It is extremely fertile in most of the places where it is found.

 

Q.11. “The dams that were constructed to control floods have triggered floods.” Analyse the statement. 

Answer: Our first Prime Minister, Mr Jawaharlal Nehru, called the dams as “the temple of modem India”. These dams that have been constructed to support the economic development of the country, can be destructive at times.

They may cause floods because sometimes, they are constructed without proper planning and also low standard construction material is used. This inferior quality of construction material increases the chances of floods. Construction of these dams can make the area in which they are constructed, ‘earthquake-prone’, which may lead to landslides and the water to flow out of dams.

 

Q.12. Name any two subjects that are included in the Concurrent List. How are laws made on these subjects? Explain.

OR

Q.12. How is the sharing of power between the Union and the State Governments basic to the structure of the Constitution of India? Explain.

Answer: Concurrent List includes subjects of common interest to both the Union and State government. These subjects are education, forest, trade unions, marriage, adoption and succession etc.

Both the state and the Union governments can make laws on these subjects. But if the laws made by both the government contradict each other, or a deadlock is created, then the law made by the Union government will prevail.

OR

Answer: Sharing of power between the Union and the State governments is very basic to the structure of the Constitution. The Constitution has distributed the legislative powers between the state government and Union government by dividing the subjects in Union list and State list, on which, these governments can make laws respectively. There is a Concurrent list as well on which, both the governments can make laws.

Also, State governments enjoy their own power in the states like Jammu and Kashmir. Many provisions of the Indian Constitution are not applicable in the states without the approval of the state government. On the other hand, the Union government enjoys its own hold over some of the union territories. This distribution of power is well embedded in the provisions of the constitution and is thus is its basic structure.

 

Q.13. “Every social difference does not lead to social division.” Justify the statement.

Answer: Social differences do not always lead to social division. They sometimes unite very different people and bring them closer by penetrating through their boundaries.

In this connection, we may take the example of the athletes Tommie Smith, Peter Norman and John Carlos who had participated in the 1968 Olympics held in Mexico. Both Smith and Carlos were African-American athletes who tried to gain international attention in the medal ceremony by wearing black gloves and raising clenched fists against racial discrimination. They attended the ceremony bare feet with black socks to represent black poverty. Their demonstration was to symbolise black power. Peter Norman was an Australian but still, he supported his co-athletes and wore a human rights badge during the ceremony. This shows that social difference does not always lead to social divisions.

 

Q.14. How can caste take several forms in politics? Explain with examples. 

Answer: Caste is considered to be the sole basis of the social community. People belonging to the same caste belong to a natural social community and have the same interests which they share amongst themselves and no one else. Caste can take various forms in politics.

(i) Caste composition of the electorate is always kept in mind when the nominations are decided by the party during elections. They tend to nominate candidates of different castes so as to muster the necessary support to win elections. When governments are formed, the parties make sure that these candidates of different castes find a place in the setup. Political parties are known to favour some castes and even are recognized as representatives of these castes. This brings prejudice and biases in terms of decisions, ideologies and other such important matters.

(ii) Universal Adult franchise has helped in compelling the political parties to mobilize and have an inclusive approach towards the castes that were earlier ignored by them. However, the inclusion of caste in politics has brought unnecessary violence and controversies. Parties try to favour certain caste and in this way, secure vote bank. Parties also incite people on the pretext of casteism and thus create disasters.

 

Q.15. “Crude oil reserves are limited all over the world. If people continue to extract it at the present rate, the reserves would last only 35 – 40 years more” Explain any three ways to solve this problem. 

Answer: Crude oil is a non- renewable resource of energy. It takes millions of years for the formation of this fuel, because of to which, it must be used judiciously. This type of fuel is being used faster than they are being produced. This causes depletion and scarcity of crude oil. 

Steps which can be taken to conserve this non-renewable source of energy are:

  • Use of public transport like buses and trains instead of self-owned vehicles will help to conserve petroleum. Carpooling will reduce the consumption of this fuel and thus scarcity will be dealt better with.
  • Use of cycles wherever possible instead of using motorbike or car.
  • Waxing floors with beeswax instead of petroleum-based commercial wax can also be beneficial.

 

Q.16. Why is it necessary to increase a large number of banks mainly in rural areas? Explain. 

OR

Q.16. Why are service conditions of formal sector loans better than informal sector? Explain. Answer: It is important to open more banks in rural areas as the formal credit sector is missing. The practice of borrowing from an informal sector that exists in rural areas, for example, local money lenders, has a number of disadvantages.

The informal sector charges high rate of interest. Informal sector makes loans very expensive as there are no external organizations controlling the credit activities of lenders.

Informal sector involves a high degree of risk as there are no proper set of rules for repayment and there is a lot of exploitation of the poor farmers. 

Lenders may exploit the borrowers, they may engage in threats and intimidation to ensure repayment of loans. There is no written agreement between the lender and the borrower. There is no legal recourse in case of informal sources of credit.

OR

Answer: Formal sector: 

  • This sector is mainly supervised by the RBI.
  • It includes banks and cooperatives and thus every clause is in writing and very clear.</li><li>In this, collateral is required.
  • It provides loans comparatively at lower rates.
  • It doesn’t lead to a debt trap.

Informal sector:

  • No external organisation supervises this sector.
  • The lenders are mainly money lenders, friends, relatives, traders and landowners etc.
  • Collateral is not required, thus it involves risk.
  • This sector charges higher interest rates without any rules or regulation.
  • This could lead to a debt trap.

 

Q.17. “How can the Government of India play a major role to make globalization fairer? Explain with examples. 

OR

Q.17. How has globalization affected the life of Indians? Explain with examples.

Answer: Fair globalization would create equal opportunities for all and would ensure that the benefits of globalization are shared better. The government can play a major role in making this possible. The policies of the government must protect the interests of all the people of the country, not only of the rich and powerful. Hence, the government can play a functional role in helping to bridge the gap between the two.

It is necessary for developing countries to have stronger trade and investment rules. They should negotiate at the WTO for fairer rules and regulations.

OR

Answer: Globalization has contributed to the booming of the Indian economy in the following ways:

  • Greater competition among producers resulting from globalisation is a great advantage to consumers as there is greater choice regarding every product before them.
  • Due to globalisation, many MNCs have increased their investments in India, this has not only helped in the inflow of capital but also helped largely in employment generation.
  • Local companies supplying raw materials to the industries that have been set as a result of globalization have prospered by leaps and bounds.
  • Large Indian companies have emerged as multinational companies. This has helped our country to increase bur contacts around the world. Globalisation has helped increase our GDP and per capita income, thus making the living standards better across the globe.

 

Q.18. How are consumers enjoying the ‘right to be informed’ in their daily life? Explain with examples.

 

Section – C

Q.19. How had the ‘First World War created economic problems in India? Explain with examples.

OR

Q.19. How had a variety of cultural processes developed a sense of collective belongingness in India during the 19th century? Explain with examples.

Answer: The economic effects of the First World War were:

  • The First World War led to huge expenditures in defence. These expenditures were to be financed by increasing the taxes and by raising customs duties.
  • During the time of the First World War, crop failure resulted in acute shortage of food.
  • During the war, the food prices increased, they almost doubled between 1913 and 1918. This increased the hardships of the people of India.
  • Villages were called upon to supply soldiers. At some rural places, the colonial government forced people to join the army. It caused widespread resentment and anger amongst the people. It set the stage for the Great Depression.
  • There was spread of influenza epidemic which contributed to the hardships of the people. The war weakened the gold standard.

OR

Answer: Nationalism spreads when people begin to believe that they are all part of the same nation and when they discover some unity, it binds them together. This sense of collective belonging unites people of different communities, regions or languages by the experience of many united struggles.

There were also a variety of cultural processes through which nationalism captured people’s imagination. History and fiction, folklore, and songs, popular poems and symbols, all played a vital role in the awakening of the spirit of nationalism. The identity of a nation is often symbolised by a figure or image. It was in the early 19th century, with the growth of nationalism that the identity of India came to be visually associated with the image of Bharat Mata. The image was first created by Bankim Chandra Chattopadhyay and in the 1870s he wrote ‘Vande Mataram’ as a hymn to the motherland. Moved by Swadeshi movement, Abanindranath Tagore painted his famous image of Bharat Mata. In this painting, Bharat Mata is portrayed as an ascetic figure, she is calm, composed, divine and spiritual.

Ideas of nationalism also developed through a movement to revive Indian folklore. In the late 19th century India, nationalists began recording folk tales sung by bards and they toured villages to gather folk songs and legends.

These tales gave a true picture of traditional culture that had been corrupted and damaged by outside forces. When people would hear these songs, they would be filled with a spirit of belongingness to the country. They felt energised and highly patriotic. It was thus, essential to spread this folk tradition in order to discover citizen’s national identity and restore a sense of pride for their past.

 

Q.20. Describe the role of ‘technology’ in the transformation of the world in the nineteenth century. 

OR

Q.20. Describe the life of workers during the nineteenth century in England.

OR

Q.20. Describe various steps taken to clean up London in the nineteenth century.

Answer: The making of modern Global world was characterized by major discoveries and inventions. Technological inventions helped developing in these ways:

  • Railways, steamships, telegraphs transformed the trade and led to easy transportation of goods and raw materials.
  • Technological advancements stimulated the process of industrialization, which expanded the production of goods and trade.
  • Refrigerated ships made transportation of perishable products, like meat, over long distances easy.
  • There was also the development of the Printing Press that lead to print revolution.
  • Communication was made easy with the invention of telephones, computers and other things like cabels, network towers etc.

OR

Answer: The life of the workers in the 19th century was miserable. They were given lower wages and were made to work for longer hours. This was the reason poverty was more prominent in cities as compared to villages. They had to work in the factories and the working environment was hazardous. They dealt with the machines without proper training and education, which was dangerous.

People from countryside rushed to cities in search of new jobs. Only a few of those, whose friends and relatives were already working into the factory could get jobs. The living conditions were so pathetic that it was expected of such people to die in a workhouse, hospital of lunatic asylum rather than in some decent working areas. Nearly 1 million Londoners (about one-fifth of the population of London at the time_ were very poor and living in un-habitable conditions.

The over-congestion was leading to epidemic diseases in the whole city. There was an urgent need to increase the number of rooms these labourers were living in. There was no proper drinking water available sometimes. The life expectancy of these poor people was nearly 29 Years of age while it was near about 55 years of age for the middle and upper-class people.

OR

Answer: The widespread use of coal in homes and industries in 19th century England raised serious problems. Shopkeepers, home owners and others complained about black fog, grey skies and black vegetation. All these factors caused bad temper, smoke related disorders and dirty clothes.

  • Congestion in the city also led to a desire for clean air. Therefore, attempts were made to decongest the localities, make the open places greener, reduce pollution and make the city more beautiful.
  • Large clocks of apartments were built and methods of control as introduced to ease the impact of an excessive housing shortage.
  • A new garden city of new Earswick was made with common gardens, beautiful views where people would live and work. Architects made efforts to plan a green city with a larger number of green belts and gardens.
  • Between the world wars, a large number of houses—most of them single-family cottages were built for the working class.

A million new houses were built and people were encouraged to live in them.

 

Q.21. Name the two major beverage crops grown ‘.in India. Describe their growing areas. 

Answer: Tea and Coffee are the two most important beverage crops of India.

Assam is the major tea producing state in India along with West Bengal and Tamil Nadu. The cropping season in Assam begins as early as March and extends almost to mid-December. Besides, the popular black tea, Assam also produces small quantities of white and green tea. This state has favourable conditions for the growth of tea. The tea plant grows well in tropical and subtropical climates. It requires deep and fertile well-drained soil, rich in humus and organic matter. Tea bushes require moist, frost-free and warm climate all through the year with abundant skilled labour. Frequent evenly distributed showers over the year ensure continuous growth of tender tea leaves.

The following are the conditions required for Tea cultivation:

Temperature: 10-30 degrees Celsius Rainfall- average yearly rainfall of 200 cm Altitude- ground level of between 600-2000 meters above sea level.

Coffee is a tropical plant which is also grown in a semi-tropical climate. The coffee tree requires heat, humidity and abundant rainfall. Karnataka, the largest coffee-producing state has favourable conditions necessary for coffee cultivation.

The temperature of the place is 23°C to 28°C. Growth is most rapid during hot rainy season and during cool dry season, berries ripen and get ready for picking. Bright sunshine and warm weather are necessary for harvesting.

It needs rainfall between 60-85 inches. Water stagnation is very harmful for coffee plants; therefore, hill slopes are best suitable for growing it.

Soil is the guiding factor in coffee plantation. The ideal soil is one with a good sub-surface drainage, and one that is easily workable. The presence of humus and other nitrogenous matter in the soil is an advantage.

 

Q.22. How can the industrial pollution of fresh water be reduced? Explain various ways. 

Answer: Main causes of water pollution is due to the wastes discharged from factories, refineries into the water bodies. These wastes contain harmful chemicals such as alkalis, acids etc. and toxic metals like mercury, lead, arsenic etc. which kill the aquatic life.

The following steps can be taken to reduce the industrial pollution:

  • Restructuring the manufacturing processes to reduce or eliminate pollutants, like, lead, zinc, arsenic through a process called Pollution Prevention. Chimneys for treating of gaseous waste are also important.
  • It is necessary to encourage industries to promote “green” methods of production and products. It includes environment-friendly operating processes.
  • It is advisable to create cooling ponds which are man made and designed to cool the heated waters of industries by evaporation, condensation and radiation.
  • It is very important to attach water treatment plant to the industry for filtration of the sewage before it enters the water bodies. Sewage treatment plants are important for treatment of polluted water.

 

Q.23. “Democracies do not appear to be very successful in reducing economic inequalities.” Justify the statement. 

OR

Q.23. “Democracy is a better form of government than any other form of government.” Justify the statement.

Answer: In most of the democracies, a small number of ultra-rich group of people enjoy a highly disproportionate share of wealth and income. The share of rich class is increasing, whereas those who are at the bottom of the society, have very little to depend upon.

Even in India, the poor constitute a large proportion of the voters and no party will like to lose their votes. Yet, democratically elected government does not appear to be keen on addressing the question of poverty as is expected of them. The situation is much worse in some other countries, like in Bangladesh more than half of its population live in poverty. People in several poor countries are now dependent on the rich countries even for basic food supplies.

Democracies are based on political equality. All citizens have equal rights in electing representatives, but this is not so in the economic field. Economic equality comes by the equitable distribution of wealth, but this is not true in democracy. The poor are becoming poorer, and sometimes they find it difficult to even meet the basic needs of life like food, shelter, health and education. There can be many factors that are prevailing in a country that make it incapable to bring about equitable distribution of wealth.

Large population: Rise in population leads to rise in family size. But, because the family income is less the people have to adjust and manage with meagre pay.

Unemployment: Because of the population explosion, the number of job opportunities are very less compared to the people. A ‘ large number of still educated people are without jobs.

Vicious circle of poverty: Poor people still have to be dependent on money-lenders to fulfill their basic needs as their income doesn’t substitute their needs.

Low literacy rate: Education is still considered to be a dream for many.

All of these factors make it difficult for a democratic government to function and work efficiently.

OR

Answer: Democracy is better than other forms of government because:

  • People are their own masters. In a democracy, every individual has a right to vote and choose his representatives in the government. Thus, it is more representative and popular.
  • The government is of the people and the laws are made by the people (or the chosen representatives) in the government. Laws are made to protect the liberty and freedom of the people. Thus, the laws are popular opinion of the citizens on the whole.
  • In a democracy, no particular religion, region, race or language is given special preference.

All individuals are given equal rights and freedom, and there is no discrimination.

  • The government is not by force. The opposition parties are allowed to criticise the government. Hence, there is a system of checks and balances in the form of democratic government.
  • Since every individual is given equal rights, there is less danger of conflicts in society. There are less conflicts based on caste, religion or region and less social tensions in society. Equitable distribution of opportunities is encouraged.

 

Q.24. What is a political party? Explain any four characteristics of a political party.

Answer: Political party is an organised group of people having a common ideology and its aim is to contest elections and come to power.

Four characteristics/features of political parties are given below:

  • Political parties seek control over the government through the process of election.
  • Parties run the government. They ensure that a country is governed as per set ideologies.
  • Parties frame their own policies in the form of manifestos which includes their vision on the basis of which they would establish governance in the country.
  • Political parties make laws and policies for the country. Members of the legislature belong to various political parties and are guided by party ideologies.
  • Parties give representation to diverse interests in society and give recognition to minorities.
  • A political party has a leader, active members and followers who support the party. (Any four)

 

Q.25. Compare the economic activities of the private sector with that of the public sector. 

Answer: 

S.No. Private Sector  Public Sector
1. Ownership of assets and delivery of services is in the hands of private individuals or companies. The govt owns most of the assets and provides all services.
2. Their main motive is to earn a profit. Their main motive is public welfare rather than to earn a profit.
3. The decision regarding production and distribution are taken by managers or owners of the company. The decision regarding production and distribution are taken by the government.
4. Due to the movie of earning a profit, it does not invest funds to construct infrastructures for public utility/facility. Due to motives of public welfare, it invests fund to construct infrastructures for public utility/ facility, like the construction of road, bridges, etc
5. Examples: Tata iron and steel company Ltd. (TISCO), Reliance industries Ltd., Etc. Examples: Railways, post office, police station, etc.

 

Q.26. (A) Two features ‘a’ and ‘b’ are marked on the given political outline map of India. Identify these features with the help of the following information and write their correct names on the lines marked near them: 

(a) The place where the Indian National Congress Session was held.

(b) The place where Gandhiji violated the salt law. 

(B) Locate and label any three of the following with appropriate symbols on the same given outline political map of India:

(i) Bokaro – Iron and Steel Plant

(ii) Gandhinagar – Software Technology Park

(iii) Tarapur – Nuclear Power Plant

(iv) Salal – Dam

(v) Tuticorin – Seaport

Answer :

Social Science 10th Previous Year Question Paper 2019 SET-II (CBSE)

Social Science

Section – A

Q.1. Explain the aim to form ‘Zollverein’ a Customs Union, in 1834 in Germany. 

OR

Q.1. Explain the main reason responsible for the eruption of a major protest in Saigon Native Girls School in Vietnam in 1926. 

Answer: Zollverein was a customs union formed in 1834 at the initiative of Prussia. The union abolished tariff barriers and internal custom dues and was willing to establish free trade with neighbouring states. It reduced the number of currencies from thirty to two. Most German states joined the Zollverein.

OR

Answer: A major protest occurred in Saigon Native Girls School in 1926. This protest erupted because a Vietnamese girl sitting in one of the front seats was asked to move back and make space for a French student. She was expelled when she refused to obey the order of the Principal. Other students supporting her were also expelled by the principal who was a colon. This led to spread of open protests.

 

Q.2. Why was printing of textbooks sponsored by the Imperial State in China? 

OR

Q.2. Why did Chandu Menon give up the idea of translation of ‘English Novels’ in Malayalam?

Answer: The printing of textbooks were sponsored by the Imperial State in China because China possessed a large bureaucratic system, which recruited their personnel through Civil service examinations. That is why, textbooks were printed in large numbers to provide them study material.

OR

Answer: The readers in Kerala were not familiar with the lifestyle of the characters portrayed in english Novels. Direct translation of english novels into Malayalam was thus making the novels boring because the readers could not relate. Thus, Chandu menon gave up the idea of translating them and wrote a story in Malayalam instead.

 

Q.3. How has Shillong solved the problem of acute shortage of water? 

OR

Q.3. How has Tamil Nadu solved the problem of acute shortage of water ?

Answer: Shillong has been able to deal with the problem of acute shortage of water by setting up Bamboo drip irrigation systems.

OR

Answer: Tamil Nadu has been able to deal with the problem of acute shortage of water by adopting Rooftop water harvesting techniques.

 

Q.4. Explain the meaning of ‘Majoritarianism’ as practised in Sri Lanka. 

Answer: The dominance of the majority community to rule the country in whichever way it wants totally disregarding the wishes and needs of minority community is known as Majoritarianism. In 1956, an act was passed by the Sinhala government to recognise Sinhala as the official language in Sri Lanka. The Sinhala government gave preferences to the Sinhalese in getting jobs and other such benefits. The Sinhalas completely disregarded the interests of Srilankan Tamils while this act was passed. This is a case of Majoritarianism.

 

Q.5. What may be a developmental goal of farmers who depend only on rain for growing crops?

OR

Q.5. What may be a developmental goal of urban unemployed youth?

Answer: The developmental goal for a farmer who depends only on rain for irrigating his crops might be to have access to better water harvesting and irrigation techniques.

OR

Answer: The developmental goal for an urban unemployed youth would be to get a decent job suitable to his/her qualifications and skills.

 

Q.6. Give anyone example of the methods to make a payment without using cash. 

Answer: Using debit card to book movie tickets, or using Paytm to purchase bread and milk packet from grocery shop is one example of a cashless transaction.

 

Q.7. Which court deals with cases involving claims up to 20 lakhs under COPRA?

 

Section -B

Q.8. Describe the great economic hardship that prevailed in Europe during the 1930s. 

OR

Q.8. Describe the serious problem faced by the modem part of Hanoi in 1903.

Answer: Great economic hardships were faced by the people of Europe in the 1930s. Some of the difficulties that they faced are:

  • The ratio of the rise of the population was larger than that of employment generation. People from rural areas were migrating to cities in search of employment, which was not as easily available because of overcrowding.
  • Small producers in towns (especially textile-producing industries) were often overthrown by the cheap machines. They faced stiff competition from imports from England.
  • Peasants still suffered under the burden of feudal dues and obligations in some regions of Europe. Rise of food prices and unemployment led to widespread pauperism in the country.

OR

Answer: The French sought to create a modem Vietnam and decided to rebuild Hanoi. The latest ideas about architecture and modem engineering skills were employed to build a new and ‘modem’ city.

(i) However, in 1903, the modem part of Hanoi was struck by bubonic plague.

(ii) The French part had wide avenues and well-planned sewer system, while the native quarter was not provided with any modem facilities. Thus, the refuse from the old city drained straight out into the river or, during heavy rains of floods, overflowed into the streets which become the cause of the plague.

(iii) Large sewers became a breeding ground for rats which began to enter the well-cared-for homes of the French through the sewage pipes, followed by massive rat hunt programme. This deteriorated the conditions. The whole city was under severe influence of this plague.

 

Q.9. How had the printing press created a new culture of reading in Europe? Explain with examples. 

OR

Q.9. How did Charles Dickens depicted the terrible effects of industrialisation on people’s lives and characters? Explain with examples.

Answer: With the introduction of the Printing Press, a new wave of print culture began in Europe. It was defined by accelerated production of books and printed material. The mass production of books lead to decrease in the prices of the books and their circulation increased. The reading culture was not restricted only to the elites but now, even the common people began to have easy access to these books.

Printers also focused on publishing the folk tales and ballads, well illustrated with pictures so that the books could be enjoyed even by a less educated audience of the villages. The books gave an opportunity to more and more people to come in contact with the ideas of philosophers and leading thinkers of the time. Thus, this changed the reading culture of Europe widely.

OR

Answer: Hard times was the tenth novel written by Charles Dickens. It was published in 1854. The process of industrialization and its effect on the labour force was the main theme of this novel. Set in the backdrop of a fictitious industrial city of Coketown, the novel sketches the condition of the then cities that were full of machinery, chimneys and smoke. The labours of the industries were considered to be ‘hands’ of the industries.

The economic pressures of the time had reduced human beings to mere instruments of production. One of his other novels, Oliver Twist also revolves around the same theme of consequences of industrialisation. Thus, Dickens proved that the prevailing idea of Utilitarianism which believed in the “the greatest amount of happiness for the greatest number of people” actually lead to the misery of several other or the happiness of the influential lead to the misery of the labourers.

 

Q.10. Describe any three main features of ‘Alluvial soil’ found in India.

OR

Q.10. Describe any three main features of ‘Black soil’ found in India.

Answer: Major characteristics of Alluvial Soil are:

  • Alluvial soil is considered as one of the most fertile soils. Alluvial soil covers the entire northern plains in India.
  • Alluvial soil contains sand, silt and clay mainly due to silt deposited by Indo-Gangetic-Brahmaputra rivers.

According to age, it is classified into Bangar (old alluvial) and Khadar (new alluvial).

  • Alluvial soil contains an ample amount of potash, phosphoric acid and lime. This soil is ideal for the growth of crops like sugarcane, wheat and rice etc.

OR

Answer: Major characteristics of Black soil are:

  • Black soil is fine textured and clayey in nature.
  • Black soil has high amount of lime, iron, magnesium and generally low quantities of ‘Phosphorus, Nitrogen and organic matter.
  • It is formed from weathered lava rocks, thus is black in colour.
  • It has a high clay content and therefore is highly retentive of water. It is extremely fertile in most of the places where it is found.

 

Q.11. Analyse the merits of Multipurpose projects. 

Answer: Merits of multipurpose projects are given below:

  • Dams are built for generating hydroelectricity.
  • Canals are made for irrigation purpose.
  • Canals can also be used for inland navigation.
  • Water supply can be used for domestic and industrial purposes.

 

Q.12. Name any two subjects that are included in the Concurrent List. How are laws made on these subjects? Explain.

OR

Q.12. How is the sharing of power between the Union and the State Governments basic to the structure of the Constitution of India? Explain.

Answer: Concurrent List includes subjects of common interest to both the Union and State government. These subjects are education, forest, trade unions, marriage, adoption and succession etc.

Both the state and the Union governments can make laws on these subjects. But if the laws made by both the government contradict each other, or a deadlock is created, then the law made by the Union government will prevail.

OR

Answer: Sharing of power between the Union and the State governments is very basic to the structure of the Constitution. The Constitution has distributed the legislative powers between the state government and Union government by dividing the subjects in Union list and State list, on which, these governments can make laws respectively. There is a Concurrent list as well on which, both the governments can make laws.

Also, State governments enjoy their own power in the states like Jammu and Kashmir. Many provisions of the Indian Constitution are not applicable in the states without the approval of the state government. On the other hand, the Union government enjoys its own hold over some of the union territories. This distribution of power is well embedded in the provisions of the constitution and is thus is its basic structure.

 

Q.13. “Every social difference does not lead to social division.” Justify the statement.

Answer: Social differences do not always lead to social division. They sometimes unite very different people and bring them closer by penetrating through their boundaries.

In this connection, we may take the example of the athletes Tommie Smith, Peter Norman and John Carlos who had participated in the 1968 Olympics held in Mexico. Both Smith and Carlos were African-American athletes who tried to gain international attention in the medal ceremony by wearing black gloves and raising clenched fists against racial discrimination. They attended the ceremony bare feet with black socks to represent black poverty. Their demonstration was to symbolise black power. Peter Norman was an Australian but still, he supported his co-athletes and wore a human rights badge during the ceremony. This shows that social difference does not always lead to social divisions.

 

Q.14. “Our society is still a male-dominated society.” Explain the statement with the help of examples.

Answer: India is considered to be a patriarchal society because men are in authority over women in all aspects of society.

(i) Political participation of women in India is very less when compared to other countries. Only 10% of the total members in Lok sabha are women. The situation is worse in state assemblies where only 5% of the total members are women. The Women reservation bill to encourage the participation of women is still pending since the past decade.

(ii) Women have entered into every field but they are still paid less than their male counterparts. The proportion of women in highly paid jobs is still less and studies have shown that on average they work more than men and yet paid less. Although the Equal Remuneration Act provides provisions for equal wages should be paid for equal work.

(iii) The literacy rate amongst women is also low as compared to men. The literacy rate is only 54% as compared to 76% among men. This shows the discrimination women have to face. Men are still considered to be the head of the family. A lot of dowry issues still emerge everywhere. Men are known to dominate women in every field using strength as a factor. Women are made to stay quiet even in cases of rapes, betrayal etc.

 

Q.15. “Crude oil reserves are limited all over the world. If people continue to extract it at the present rate, the reserves would last only 35 – 40 years more” Explain any three ways to solve this problem. 

Answer: Crude oil is a non- renewable resource of energy. It takes millions of years for the formation of this fuel, because of to which, it must be used judiciously. This type of fuel is being used faster than they are being produced. This causes depletion and scarcity of crude oil. 

Steps which can be taken to conserve this non-renewable source of energy are:

  • Use of public transport like buses and trains instead of self-owned vehicles will help to conserve petroleum. Carpooling will reduce the consumption of this fuel and thus scarcity will be dealt better with.
  • Use of cycles wherever possible instead of using motorbike or car.
  • Waxing floors with beeswax instead of petroleum-based commercial wax can also be beneficial.

 

Q.16. Why is it necessary to increase a large number of banks mainly in rural areas? Explain. 

OR

Q.16. Why are service conditions of formal sector loans better than informal sector? Explain. Answer: It is important to open more banks in rural areas as the formal credit sector is missing. The practice of borrowing from an informal sector that exists in rural areas, for example, local money lenders, has a number of disadvantages.

The informal sector charges high rate of interest. Informal sector makes loans very expensive as there are no external organizations controlling the credit activities of lenders.

Informal sector involves a high degree of risk as there are no proper set of rules for repayment and there is a lot of exploitation of the poor farmers. 

Lenders may exploit the borrowers, they may engage in threats and intimidation to ensure repayment of loans. There is no written agreement between the lender and the borrower. There is no legal recourse in case of informal sources of credit.

OR

Answer: Formal sector: 

  • This sector is mainly supervised by the RBI.
  • It includes banks and cooperatives and thus every clause is in writing and very clear.
  • In this, collateral is required.
  • It provides loans comparatively at lower rates.
  • It doesn’t lead to a debt trap.

Informal sector:

  • No external organisation supervises this sector.
  • The lenders are mainly money lenders, friends, relatives, traders and landowners etc.
  • Collateral is not required, thus it involves risk.
  • This sector charges higher interest rates without any rules or regulation.
  • This could lead to a debt trap.

 

Q.17. “How can the Government of India play a major role to make globalization fairer? Explain with examples. 

OR

Q.17. How has globalization affected the life of Indians? Explain with examples.

Answer: Fair globalization would create equal opportunities for all and would ensure that the benefits of globalization are shared better. The government can play a major role in making this possible. The policies of the government must protect the interests of all the people of the country, not only of the rich and powerful. Hence, the government can play a functional role in helping to bridge the gap between the two.

It is necessary for developing countries to have stronger trade and investment rules. They should negotiate at the WTO for fairer rules and regulations.

OR

Answer: Globalization has contributed to the booming of the Indian economy in the following ways:

  • Greater competition among producers resulting from globalisation is a great advantage to consumers as there is greater choice regarding every product before them.
  • Due to globalisation, many MNCs have increased their investments in India, this has not only helped in the inflow of capital but also helped largely in employment generation.
  • Local companies supplying raw materials to the industries that have been set as a result of globalization have prospered by leaps and bounds.
  • Large Indian companies have emerged as multinational companies. This has helped our country to increase bur contacts around the world. Globalisation has helped increase our GDP and per capita income, thus making the living standards better across the globe.

 

Q.18. How are consumers enjoying the ‘right to be informed’ in their daily life? Explain with examples.

 

Section – C

Q.19. How had the ‘First World War created economic problems in India? Explain with examples.

OR

Q.19. How had a variety of cultural processes developed a sense of collective belongingness in India during the 19th century? Explain with examples.

Answer: The economic effects of the First World War were:

  • The First World War led to huge expenditures in defence. These expenditures were to be financed by increasing the taxes and by raising customs duties.
  • During the time of the First World War, crop failure resulted in acute shortage of food.
  • During the war, the food prices increased, they almost doubled between 1913 and 1918. This increased the hardships of the people of India.
  • Villages were called upon to supply soldiers. At some rural places, the colonial government forced people to join the army. It caused widespread resentment and anger amongst the people. It set the stage for the Great Depression.
  • There was spread of influenza epidemic which contributed to the hardships of the people. The war weakened the gold standard.

OR

Answer: Nationalism spreads when people begin to believe that they are all part of the same nation and when they discover some unity, it binds them together. This sense of collective belonging unites people of different communities, regions or languages by the experience of many united struggles.

There were also a variety of cultural processes through which nationalism captured people’s imagination. History and fiction, folklore, and songs, popular poems and symbols, all played a vital role in the awakening of the spirit of nationalism. The identity of a nation is often symbolised by a figure or image. It was in the early 19th century, with the growth of nationalism that the identity of India came to be visually associated with the image of Bharat Mata. The image was first created by Bankim Chandra Chattopadhyay and in the 1870s he wrote ‘Vande Mataram’ as a hymn to the motherland. Moved by Swadeshi movement, Abanindranath Tagore painted his famous image of Bharat Mata. In this painting, Bharat Mata is portrayed as an ascetic figure, she is calm, composed, divine and spiritual.

Ideas of nationalism also developed through a movement to revive Indian folklore. In the late 19th century India, nationalists began recording folk tales sung by bards and they toured villages to gather folk songs and legends.

These tales gave a true picture of traditional culture that had been corrupted and damaged by outside forces. When people would hear these songs, they would be filled with a spirit of belongingness to the country. They felt energised and highly patriotic. It was thus, essential to spread this folk tradition in order to discover citizen’s national identity and restore a sense of pride for their past.

 

Q.20. Describe the role of ‘technology’ in the transformation of the world in the nineteenth century. 

OR

Q.20. Describe the life of workers during the nineteenth century in England.

OR

Q.20. Describe various steps taken to clean up London in the nineteenth century.

Answer: The making of modern Global world was characterized by major discoveries and inventions. Technological inventions helped developing in these ways:

  • Railways, steamships, telegraphs transformed the trade and led to easy transportation of goods and raw materials.
  • Technological advancements stimulated the process of industrialization, which expanded the production of goods and trade.
  • Refrigerated ships made transportation of perishable products, like meat, over long distances easy.
  • There was also the development of the Printing Press that lead to print revolution.
  • Communication was made easy with the invention of telephones, computers and other things like cabels, network towers etc.

OR

Answer: The life of the workers in the 19th century was miserable. They were given lower wages and were made to work for longer hours. This was the reason poverty was more prominent in cities as compared to villages. They had to work in the factories and the working environment was hazardous. They dealt with the machines without proper training and education, which was dangerous.

People from countryside rushed to cities in search of new jobs. Only a few of those, whose friends and relatives were already working into the factory could get jobs. The living conditions were so pathetic that it was expected of such people to die in a workhouse, hospital of lunatic asylum rather than in some decent working areas. Nearly 1 million Londoners (about one-fifth of the population of London at the time_ were very poor and living in un-habitable conditions.

The over-congestion was leading to epidemic diseases in the whole city. There was an urgent need to increase the number of rooms these labourers were living in. There was no proper drinking water available sometimes. The life expectancy of these poor people was nearly 29 Years of age while it was near about 55 years of age for the middle and upper-class people.

OR

Answer: The widespread use of coal in homes and industries in 19th century England raised serious problems. Shopkeepers, home owners and others complained about black fog, grey skies and black vegetation. All these factors caused bad temper, smoke related disorders and dirty clothes.

  • Congestion in the city also led to a desire for clean air. Therefore, attempts were made to decongest the localities, make the open places greener, reduce pollution and make the city more beautiful.
  • Large clocks of apartments were built and methods of control as introduced to ease the impact of an excessive housing shortage.
  • A new garden city of new Earswick was made with common gardens, beautiful views where people would live and work. Architects made efforts to plan a green city with a larger number of green belts and gardens.
  • Between the world wars, a large number of houses—most of them single-family cottages were built for the working class.

A million new houses were built and people were encouraged to live in them.

 

Q.21. Name the two major beverage crops grown ‘.in India. Describe their growing areas. 

Answer: Tea and Coffee are the two most important beverage crops of India.

Assam is the major tea producing state in India along with West Bengal and Tamil Nadu. The cropping season in Assam begins as early as March and extends almost to mid-December. Besides, the popular black tea, Assam also produces small quantities of white and green tea. This state has favourable conditions for the growth of tea. The tea plant grows well in tropical and subtropical climates. It requires deep and fertile well-drained soil, rich in humus and organic matter. Tea bushes require moist, frost-free and warm climate all through the year with abundant skilled labour. Frequent evenly distributed showers over the year ensure continuous growth of tender tea leaves.

The following are the conditions required for Tea cultivation:

  1. Temperature: 10-30 degrees Celsius Rainfall- average yearly rainfall of 200 cm Altitude- ground level of between 600-2000 meters above sea level.
  2. Coffee is a tropical plant which is also grown in a semi-tropical climate. The coffee tree requires heat, humidity and abundant rainfall. Karnataka, the largest coffee-producing state has favourable conditions necessary for coffee cultivation.
  3. The temperature of the place is 23°C to 28°C. Growth is most rapid during hot rainy season and during cool dry season, berries ripen and get ready for picking. Bright sunshine and warm weather are necessary for harvesting.
  4. It needs rainfall between 60-85 inches. Water stagnation is very harmful for coffee plants; therefore, hill slopes are best suitable for growing it.
  5. Soil is the guiding factor in coffee plantation. The ideal soil is one with a good sub-surface drainage, and one that is easily workable. The presence of humus and other nitrogenous matter in the soil is an advantage.

 

Q.22. How can the industrial pollution of fresh water be reduced? Explain various ways. 

Answer: Main causes of water pollution is due to the wastes discharged from factories, refineries into the water bodies. These wastes contain harmful chemicals such as alkalis, acids etc. and toxic metals like mercury, lead, arsenic etc. which kill the aquatic life.

The following steps can be taken to reduce the industrial pollution:

  • Restructuring the manufacturing processes to reduce or eliminate pollutants, like, lead, zinc, arsenic through a process called Pollution Prevention. Chimneys for treating of gaseous waste are also important.
  • It is necessary to encourage industries to promote “green” methods of production and products. It includes environment-friendly operating processes.
  • It is advisable to create cooling ponds which are man made and designed to cool the heated waters of industries by evaporation, condensation and radiation.
  • It is very important to attach water treatment plant to the industry for filtration of the sewage before it enters the water bodies. Sewage treatment plants are important for treatment of polluted water.

 

Q.23. “Democracies do not appear to be very successful in reducing economic inequalities.” Justify the statement. 

OR

Q.23. “Democracy is a better form of government than any other form of government.” Justify the statement.

Answer: In most of the democracies, a small number of ultra-rich group of people enjoy a highly disproportionate share of wealth and income. The share of rich class is increasing, whereas those who are at the bottom of the society, have very little to depend upon.

Even in India, the poor constitute a large proportion of the voters and no party will like to lose their votes. Yet, democratically elected government does not appear to be keen on addressing the question of poverty as is expected of them. The situation is much worse in some other countries, like in Bangladesh more than half of its population live in poverty. People in several poor countries are now dependent on the rich countries even for basic food supplies.

Democracies are based on political equality. All citizens have equal rights in electing representatives, but this is not so in the economic field. Economic equality comes by the equitable distribution of wealth, but this is not true in democracy. The poor are becoming poorer, and sometimes they find it difficult to even meet the basic needs of life like food, shelter, health and education. There can be many factors that are prevailing in a country that make it incapable to bring about equitable distribution of wealth.

Large population: Rise in population leads to rise in family size. But, because the family income is less the people have to adjust and manage with meagre pay.

Unemployment: Because of the population explosion, the number of job opportunities are very less compared to the people. A ‘ large number of still educated people are without jobs.

Vicious circle of poverty: Poor people still have to be dependent on money-lenders to fulfill their basic needs as their income doesn’t substitute their needs.

Low literacy rate: Education is still considered to be a dream for many.

All of these factors make it difficult for a democratic government to function and work efficiently.

OR

Answer: Democracy is better than other forms of government because:

  • People are their own masters. In a democracy, every individual has a right to vote and choose his representatives in the government. Thus, it is more representative and popular.
  • The government is of the people and the laws are made by the people (or the chosen representatives) in the government. Laws are made to protect the liberty and freedom of the people. Thus, the laws are popular opinion of the citizens on the whole.
  • In a democracy, no particular religion, region, race or language is given special preference.

All individuals are given equal rights and freedom, and there is no discrimination.

  • The government is not by force. The opposition parties are allowed to criticise the government. Hence, there is a system of checks and balances in the form of democratic government.
  • Since every individual is given equal rights, there is less danger of conflicts in society. There are less conflicts based on caste, religion or region and less social tensions in society. Equitable distribution of opportunities is encouraged.

 

Q.24. Explain any five functions of the ruling party. 

Answer: Functions of the Ruling Party are following:

  • The ruling party is one which has anchored dominant part in elections and has framed the legislature. Ruling party guarantees that the nation keeps running according to set belief systems and projects.
  • Ruling Party offers portrayal to different enthusiasm for society, they offer acknowledgement to minorities and give access to individuals to government machinery. The nationals can likewise vent their complaints to nearby local party leaders regarding any arrangement and its execution.
  • Party in power reassures and makes individuals mindful about its welfare plans, it encourages individuals to think about its accomplishments, its plan and profit and its advantages.
  • It establishes foreign relations and decide the foreign policy of the country. Thus the future and the reputation of the country is upheld by them.
  • Ruling Party organises schemes and campaigns for the welfare of the people. It makes schemes for the development of society both culturally and economically.

 

Q.25. Explain any five differences between organised and unorganised sectors. 

Answer: Five differences between organised and unorganised sectors are as follows:

 

 

Q.26. (A) Two features ‘a’ and ‘b’ are marked on the given political outline map of India. Identify these features with the help of the following information and write their correct names on the lines marked near them: 

(a) The place where the Indian National Congress Session was held.

(b) The place where Gandhiji violated the salt law. 

(B) Locate and label any three of the following with appropriate symbols on the same given outline political map of India:

(i) Bokaro – Iron and Steel Plant

(ii) Gandhinagar – Software Technology Park

(iii) Tarapur – Nuclear Power Plant

(iv) Salal – Dam

(v) Tuticorin – Seaport

Answer :

×

Hello!

Click one of our representatives below to chat on WhatsApp or send us an email to info@vidhyarthidarpan.com

×